Вы находитесь на странице: 1из 94

TORTS INTRODUCTION Tort = not a crime, civil lawsuit to get monetary damages to recover for a wrongfully caused injury.

y. o Intentional Torts involve intentional actions that cause injury to another. Battery- defined differently than the CRIME of battery. Assault False Imprisonment Trespass Intentional Infliction of Emotional Distress o Negligence biggest category of torts, MOST IMPORTANT Fault-based cause of action is at fault (when fails to behave as a reasonable person and causes injury to another, he becomes liable for the injuries) Unreasonable behavior that causes injuries. Slip and fall, car accident, malpractice. Typically covered by insurance. o Strict Liability liability without fault. Product liability if your product harms another, you have to pay for those damages. INTENTIONAL TORTS Intentional Tort - can recover only if the intentionally invaded the specific interest that is protected by the tort. o Torts differ from each other in that they protect different interests of the . o For each tort, the must prove the elements of the tort in order to recover. o has the burden of introducing evidence tending to prove every element in order to avoid suffering a directed verdict. o If the produces evidence on every element & the case is submitted to the jury, the has the burden of persuading the jury that every element has been met. Even if the proves every element, the can escape liability by establishing a defense. The has the burden of producing evidence & persuading the jury on these defenses. o Causation is an element in ALL tort actions. Transfer of intent- if you intend to commit an intentional tort against A, but actually harm B, the intent transfers as if you had intended to commit the tort against B. This applies solely to the five intentional torts. Intentional torts are often things we planned or thought through. Negligence happens when you didnt have a conscious or coherent plan. Purpose: the protection not just of physical integrity and harm, but also the protection of dignity associated with contact with the body, ones personal space. Children & Intentional Torts o Children CAN be held liable for intentional torts. Parents arent normally liable for the intentional torts of their children. Do you or do you not have the capacity to form intent? Children are treated as adults for the purposes of INTENTIONAL TORTS. Exceptions: in certain jurisdictions- children below the age of 7 cant be liable for intentional torts because they cant form intent. 5 is usually the lowest age we go to hold a child liable for intentional torts.
1

Advantage of a rule like this: prevents needless lawsuits against children who are bound to misbehave, and are not capable of intent. States can choose between bright-line rule or judge on a case-by-case basis to determine whether child is capable of intent (flexible standards). These occur throughout tort law, and law in general. Bright-Line Rule: Below 7= no intentional tort liability. Makes everything clear cut, equal treatment, easy to apply, saves time & resources. (Ex. In Garratt v. Dailey, if this rule applied, the judge would dismiss the case) Disadvantages = can be arbitrary at times. Flexible Standards: above 7 years old, determine on a case-by-case basis. More fair in application; judges people in their real culpability. Disadvantage = dont know in advance if the will be considered capable of forming intent or not, harder to predict outcome, have to invest time and resources. Policy is to protect children who dont yet have the capacity to form intent.

INTENTIONAL TORT - BATTERY Battery is the intentional infliction of a harmful or offensive bodily contact. What is the purpose of battery law? o Protects relief & compensation for the injuries caused due to intentional behavior (also feature of negligence law). o Reinforce socially accepted behavior (set out what norms require); dont intentionally injure someone. o Protects against indignities. o Prevent violence (deterrence = like criminal law) o Prevent private revenge & retribution (which existed prior to the establishment of criminal laws). Elements of Battery: o Act volitional muscular movement, must have done something/caused contact. o Intent must have intended harmful contact or must have known with substantial certainty that a harmful contact would result. Subjective have to get into the head of the person youre trying to determine the intent (would this specific person know what they were going to do at the time they engaged in the action?) 3 possible wasy to define intent (Ghassemieh v. Schafer): Intent to harm. Intent to contact. Intent to offend. o Harmful or Offensive Contact actually has to cause harm/injury/be offensive to the /reasonable person. Objective standard. LINK THIS ELEMENT TO LACK OF CONSENT!!!!!!! o Causation there must be a link between the act and the suffered injury. Types of Mental States for Battery: o Purpose (Intent) o Knowledge w/substantial certainty (Intent) Objective v. Subjective standards:
2

Objective: Standard by which a reasonable person would do. Offensiveness is JUDGED BY OBJECTIVE. Only way to have consistency in the law. Subjective: By what THIS SPECIFIC plaintiff or defendant knew. Intent is ALWAYS JUDGED BY A SUBJECTIVE STANDARD.

Eggshell skull: o If is found to have committed a battery upon , then is liable for all resulting damage from the battery, even if the injury is unforeseen at the time. (Set a force in motion) Ex: Kick someone playfully who has osteoporosis and the leg shatters; is fully liable for those injuries caused. ELEMENT OF INTENT o Types of INTENT: Desire to cause contact ( swings at and hits him). Desire to frighten. Even if didnt intend contact to occur and just wanted to make think it would, this intent to cause assault is enough for battery if contact ensues. ( swings at , intending to just miss s nose, but miscalculates and makes contact). Substantially certain variety of intent: if knows that a harmful or offensive contact is substantially certain to occur, the fact that doesnt desire that contact is irrelevant. Test is SUBJECTIVE: what really thought, not what he SHOULD have thought; is protected if he didnt realize the contact would occur. o Ghassemieh V. Schafer: brought charges 35 months after her former student, , pulled a chair from under her, causing her to fall and hurt her back. Statute of limitations (bright line rule) on battery had already expired (1 year). Therefore, s counsel filed a negligence action (3 years). s counsel arguing for battery because statute was up. Judge erroneously thought battery & negligence could not overlap; but they are not mutually exclusive. Jury found battery for even though she had no intent to harm. Battery can occur without the intent to HARM. lawyers objection said that battery does require the intent to HARM; but the s intent was to embarrass. However, thats enough for a battery. INTENT to cause a CONTACT of a particular sort that is offensive to a reasonable person is sufficient intent for the purpose of battery. Substantive (Black Letter Law): Intent to harm is not required for battery (intent Schafer had, which was not to HARM but to pull a prank, was sufficient to require the intent to commit a battery judges instructions/definition of battery was correct.) Relevance of (Childs) Age children can commit batteries, as long as they have the capacity to form intent (not necessarily to harm, but to form intent to cause a contact of the sort of that would be offensive to a reasonable person) Procedural: Statute of Limitations: after a certain date you cant bring a claim. These exist to resolve cases quicker, act on their rights to bring an action. Also, longer you go from the event, harder it is to figure out what exactly happened (harder to establish facts, evidence, etc.) SOLs different in each state for each thing.
3

Battery v. Negligence diff because on average, the conduct behind battery is more culpable and more directly observable, less likely to take as long to realize you have a battery claim (1 year SOL; whereas negligence 3 years). Lawyers must object to legal error when its made at trial correctly and specifically (what piece you are objecting to) most jurisdictions have this rule (not every court will apply their rule as harshly as they did in Ghassemieh). What does it mean that battery & negligence can overlap? (Ghassemieh) intended the contact (battery; risk-causing behavior), didnt necessarily intend the consequences (negligence). When you intend the contact but NOT the consequences battery & negligence overlap. Cant get a double recovery, though. If filed a suit within a year for $100,000 and was charged with battery AND negligence, she couldnt get damages for 2 different tort causes of action. She would only get $100,000. If you have 2 causes of action, do you want to pursue both? It depends on the claim you have. In the case of negligence & battery, juries think its worse to INTEND contact, so it could bolster your case. Example of negligence but not a battery not paying attention to a red light and you hit a car didnt intend the contact/consequences, but your behavior was negligent. Garratt. v. Dailey: young boy 5 years 9 months pulled a chair out from under who falls, causing injuries. Damages $11,000. Judges law of battery was incomplete; right that there was no intent, but didnt mention that the substantial certainty that the boy knew she was going to sit in the chair and fall needed to be proven. Appellate ct said they can infer did not have intent, but remanded it to make it absolutely clear (because he didnt say he had no knowledge with substantial certainty). Stretching this to find the legal issue. Appellate court says there are TWO types of intent: Purpose-type intent; or Knowledge with substantial certainty-type intent. Trial court CLEARLY found, with regard to purpose-type intent, that the boy did not MEAN (or desire) to injure her. Fairly clear he didnt even desire to play a joke on her. How can you have battery without intent to harm and intent to contact? Intent is a SUBJECTIVE STANDARD: Must look at WHAT was in his head when he moved the chair. Dont have to believe the testimony of the , but you can INFER from the testimony of others that the person did in fact have the intent. Black-Letter Law: In order that an act may be done with the intention of bringing about a harmful or offensive contact to a particular person, the act must be done with the purpose of causing the contact or with knowledge on the part of the action that such contact is substantially certain to be produced.

Purpose-Type Intent vs. Knowledge with Substantial Certainty o Is it possible to have KSC, without purpose? Yes Garratt v. Dailey. Judge found that he had KSC, even though he had no purpose. o What do you have to know for substantial certainty? What would the trial judge have had to have found in order to find that had substantial certainty?

o o

Battery would be established if, in addition to the s fall, it was proved that KNEW WITH SUBSTANTIAL CERTAINTY, WHEN HE MOVED THE CHAIR, THAT THE WOULD ATTEMPT TO SIT DOWN WHERE THE CHAIR HAD BEEN. The mere absence of any intent to injure the or to play a prank on her or to embarrass her (offend), would not absolve him from liability IF, in fact, he had such knowledge (that she would attempt to sit where the chair had been).

Intent Hypotheticals o (1) A decides she doesnt like B. Wants B to suffer. Going to pull chair from under B, hoping she hurts back. Purpose-type intent. Desired for B to be harmed. o (2) Difference between purpose & KSC: A sees B walking on beach, decides to fight B for no reason. A has a rock in hand, but has a bad arm. Wants to hurt B with rock. Throws it, and rock does hit B. KSC intent? No. Because not sure the rock was going to hit her; doubted rock would hit her. Therefore, its PURPOSE intent, which is enough to constitute battery. Can have both in some cases. o (3) A is in elevator with B, taps B on shoulder to tell them they are getting off on the floor the elevator stops on. B is hemophiliac, bleeds to death. Is A liable for battery? Does A have intent? Yes, but not in an offensive manner. Intended the contact, but Garratt says you dont have to intend the offensive part of the contact you dont have to intend the consequences, but there is no contact offensive to a reasonable person. Strict Liability (liability without fault). o (4) A thinks that no man can resist her; sees B in line and puts her arm around him. Has she committed a battery? Yes. Why? Have the purpose (to contact), and can be considered an offensive contact. Well-meaning lout hypothetical. Do you hold them responsible when they thought the contact would be welcome? Could you make a prima facie case based on these facts? (Could it go to a jury/argument be made for battery?) Yes. Violates social norms. Intended a contact that we can look at and say people dont do that because its offensive to a reasonable person enough for a battery. o (5) A is running in a race. Loses by hundredths of a second. So aggravated, stands are packed with people. In aggravation, throws cleats in stands and hits someone. Didnt want or desire to hit anyone. Knew stands were crowded. Does A have the requisite knowledge for a battery? Yes. Because its so crowded, when A threw cleats, he knew with substantial certainty that someone is going to get hit. Regardless that A didnt want to hit anyone. KSC, without purpose, constitutes battery. o (6) A goes to Gator City and is doing vodka shots all night. Gets car and drives home. Runs someone over. Did A commit a battery? Reckless (conscious disregard of a high degree of risk) or did A know with substantial certainty? Dont EXPECT to hit anyone. Had no PURPOSE didnt WANT the accident. Accidents more likely to happen when drunk can injure someone. Violation of social norms. High risk of injury. Even though theres a high degree of risk, its RECKLESSNESS, not KSC. Recklessness not enough for a battery. Substantial certainty of hitting someone isnt in this case- people drive drunk all the time.
5

Stadium hypothetical: almost impossible to throw cleats without hitting someone. Almost HAVE to, and that will be the result.

ELEMENT OF HARMFUL OR OFFENSIVE CONTACT o The contact can be either harmful or offensive. Offensive: as long as s dignity is harmed, no injury is necessary. o Contact can be by indirect means Not necessarily s person touching . (Example: throws an object at ). Mechanical devices typically involve a battery unless owner had a privilege. Example: puts a security system in car that administers electric shock to anyone who touches the car battery. o When a person exceeds a privilege, could be a battery. tries to defend himself against an attack from , uses excessive force- liable for battery. o Fisher v. Carrousel Motor Hotel, Inc.: invited to a buffet-style luncheon at the Brass Ring Club, when approached , snatched the plate from him, and shouted that he was a Negro and could not be served. Testified that highly embarrassed and hurt him in the presence of his associates. Court ruled that battery does not have to be actionable, but can be willful and cause invasion of the s person, but not actual harm, to still be a battery. Ruled for . There can be a battery without physical assault, and actual physical contact is not necessary to constitute a battery, so long as there is contact with clothing or an object closely identified with the body. Black-Letter Law: To constitute an assault and battery, it is not necessary to touch the s body or even his clothing; knocking or snatching anything from the s hand or touching anything connected with his person when done in an offensive manner, is sufficient. How far does this go? Depends on proximity. Not protecting her property right, but an invasion close to her body that invades personal space. Legal Relevance? brings battery action against , manager of the Brass Ring Club, and the Carrousel Motor Hotel which has the club in it. Want hotel in the case because the hotel has MONEY can pay damages. Vicarious liability (form of vicarious responsibility that applies to when an employer is liable= respondeat superior) responsible for the torts of your employees that they commit while acting within the scope of their employment. Frolic and Detour: If your employee is not acting within the scope of employment, you have an argument to get out of vicarious liability. What is the difference between an assault & a battery? Battery have to have contact, direct or indirect. Assault have to have fear or apprehension of immediate battery. There can be BOTH battery & assault at the same time. Many times battery is preceded by assault, but not always. Context & intent can change the entire situation. Driving in a parking lot someone bangs on your window and screams at you. Battery? Depends on what the person is saying when banging, where the person is banging. Usually dont associate car close enough with body to define battery. INTENT judged by SUBJECTIVE standard. OFFENSIVENESS judged by OBJECTIVE standard (reasonable person).
6

Hypothetical: A is in elevator with germaphobe, but doesnt know they are a germaphobe. Taps person on shoulder to tell them they are getting out of elevator. Battery? No. May be offensive to him, but not to a reasonable person. If you dont know the sensitivity of the , no battery (reasonable person standard). Hypothetical: What if A DID know he was a germaphobe and deliberately tapped him on shoulder? Battery? Yes. Because there was INTENT- you knew of the special sensitivity of the and acted just to get them. Look at knowledge of super sensitivity. Substantial certainty intent in this case. Judge if person knows the offensiveness of the contact. If you dont know of the sensitivity, you dont know the offensiveness of the contact, no way to know.

INTENTIONAL TORTS ASSAULT Assault is the intentional causing of an apprehension of harmful or offensive contact. The must prove that the intended to cause apprehension (fear) of imminent harmful or offensive bodily contact, intended to cause actual harmful or offensive bodily contact, or intended to cause a confinement. In order to have an assault claim, you have to have: o Act (volitional muscular movement) o Intent (almost same as intent for battery) o Apparent present ability o Reasonable Apprehension of Immediate Battery (Imminent harmful or offensive contact) o Causation (applies same way in negligence as it does in intentional torts) must plead & prove these elements by a preponderance of the evidence. Vetter v. Morgan: was stopped at a red light late at night when car behind her with passenger, , approached; allegedly screamed and made threatening gestures at , threatened to remove from her van, and spat on her van door. claims when light turned green, s car suddenly veered into her lane, causing her to turn sharply to the right, injuring her. o Court of Appeals decided s threat and the acts and circumstances surrounding it could put someone in s position in apprehension of imminent bodily harm. o s behavior was so extreme that could have reasonably believed he would immediately try to carry out his threat. It is not necessary that the victim be placed in apprehension of instantaneous harm. It is sufficient if it appears there will be no significant delay. o s ability to prevent the threatened harm by flight or self-defense does not preclude an assault. It is enough that believed that was capable of immediately inflicting the contact unless prevented by self-defense, flight, or intervention by others. o Black-Letter Law: Words can constitute assault if together with other acts or circumstances they put the other in reasonable apprehension of imminent harmful or offensive contact with his person. o Why do the facts of the scenario matter, arguably? Mere words cannot be assault. But words almost always come in a context.
7

o o

Have to judge what was going on in her head in light of the surrounding circumstance. Would a reasonable person in her shoes have apprehension of an immediate battery? Do all the facts ADDED TOGETHER convince a jury with a clear preponderance that there was a present ability? Was it apparent to her that they had the ability to carry out a battery with no delay? If its arguable, judge sends case to a jury to decide. Does the PACKAGE show that a reasonable person in her shoes have an apprehension that the person was going to batter her? Her fear is tied to her apprehension of immediate battery.

Intent issues in assault: o Two distinct intents, either can suffice: (1) intends to commit a battery, but fails; or (2) intends to put in apprehension, but not to really cause the contact (so that the intent is attempt to frighten) o Transferred Intent operates in assault cases. Contact o The contact must be, in s mind, imminent. must be aware of the danger BEFORE it happens; its not enough that the contact eventually happens. If did not see the contact coming in advance, this would negate assault. o s apprehension must be that there will be a contact with herself, NOT a contact with another. Example: shoots at X, while Xs mother, , looks on. If feared only that the bullet would hit X, not that it would hit herself, has not been assaulted. o must be apprehensive of a harmful or offensive contact, but not necessarily apprehensive of a BATTERY. If thinks the contact is some natural event or some unintentional human event, that can still be enough to satisfy the apprehension requirement. Example: sees a spider and thinks its real and that itll bite him. Its really a fake put there by to scare . Even though doesnt think a human was involved, and doesnt think this is an attempt at battery, its still an ASSAULT because has been put in apprehension of a harmful or offensive contact. o As with battery, consider the possibility whenever someone exceeds the scope of a privilege. Example: , a homeowner, shoots at , who knows is an unarmed burglar. Since wasnt permitted to use deadly force here, he had no defense of selfdefense, so he has committed assault. Hypotheticals: o (1) Shawn Johnson goes to Gator City. Johnson had a little bit to drink and is a little aggressive. Bouncer wont let her in. She gets in his face and says Im going to take you out. Assault? Yes. Absolutely. Reasonable apprehension of battery (can happen even if you have no fear of being harmed- just about invasions of personal space). Apprehension is different than fear. Relevant to whats apparent to HIM, to what he, in reasonable circumstances, would perceive. s right to recover is not negated by the fact that she is confident of his own ability to take action to avoid the contact.
8

(2) If I point a gun at you and say Im gonna shoot you = assault. (3) If I call you and say I am pointing a gun at your best friend, assault for you? Can fear for others form the basis of an assault claim? NO not an assault to YOU. Doesnt meet definition. Assault is narrow legal definition apprehension of personal space. DEFENSES (PRIVILEGES)/CONSENT o o Order of trial has burden of making prima facie case first, then can win by either attacking evidence of each element of battery, or can win by asserting a defense has burden of proof by preponderance of the evidence. How do you analyze/organize a case with multiple torts? o 1. Strongest claims go first. o 2. As you run out of time & space, do weaker claims. o 3. Within each claim, talk about each element of the tort in order. RIGID structure. o That is how EVERY lawyer proves battery by preponderance uses elements as a checklist. must prove in prima facie battery case a lack of consent. o Cant be offensive if it was consented to. o Lack of consent is what makes it offensive to a reasonable person has the burden of pleading & proving the lack of consent. CONSENT o has to make the defense to negate liability says he had consent. o Two ways you can have consent: (1) Actual Consent: Can prove actually, subjectively, consented. (2) Objective Manifestation of Consent: apparent consent. can reasonably rely on actions of ; which means burden of proof is on to prove lack of consent. o Why is burden on the ? Impossible for us to figure out what is going on in other peoples minds. Should be able to rely on peoples actions as a guide to our own contact to determine if certain contact will be consented to. o OBrien v. Cunard S.S. Co.: was on a ship where people who had unmarked arms were being given small pox vaccinations, and certifications of receiving them, to prevent detention upon arrival. told doctor she had been vaccinated before, but had no mark, and stood in line with other people waiting to receive shots. gave her vaccine, along with papers confirming it. She sued for battery & negligence. Didnt make it clear to doctor she didnt want the shot. Is consent subjective or objective? BOTH in one way it can be established that she showed ACTUAL (subjective), or the other way OBJECTIVE MANIFESTATION OF CONSENT (apparent). says she did not subjectively consent to the vaccine. Court says, however, she gave an objective manifestation of consent, and was entitled to rely on this. Black Letter Law: If the plaintiffs behavior was such as to indicate CONSENT on her part, he was justified in his act, whatever her unexpressed feelings may have been. In determining whether she consented, he could be guided only by her overt acts and the manifestations of her feelings. As a matter of public policy, why is doctor allowed to rely on this?
9

Places burden of consent onto receiver of care (not doctor) - vaccinations important to avoid widespread disease, and statistically some people will get sick. Protects doctor who acts reasonably and responsibly (he had absolutely no reason to believe that she did not want the shot.) Also brought as a negligence claim: failure to behave as a reasonable doctor would under the circumstances (get informed consent before performing procedure) held not negligent. In a medical malpractice context, if the doctor failed to get the patients informed consent before performing a certain procedure, he MAY be found to have battered the patient.

Overall v. Kadella: Fight breaks out after a hockey game. states that he was struck from behind (by a hockey stick) and retaliated by throwing a punch. contends that he didnt hit anyone with a hockey stick, and that skated over to the bench where the was and provoked the attack. Referees testified that poked with a hockey stick to get his attention and saw strike him. sues for knocking him unconscious and fracturing the bones around his right eye (battery). Rules of game dont automatically tell you whether you have a battery or not, has to be more than just being a rule. Rules do help set norms but not absolutely definitive. Issue: What does the court say you consent to by virtue of participation in the game? Consent is limited in scope. Game ends = consent ends. Walk off field = consent ends. Court says the law in regards to scope of consent: Participation in a game involves a manifestation of consent to those bodily contacts which are permitted by the rules of the game. However, there is general agreement that an intentional act causing injury, which goes beyond what is ordinarily permissible, is an assault & battery for which recovery may be had. Black-Letter Law: An intentional act causing injury, which goes beyond what is ordinarily permissible, is an assault and battery for which recovery may be had. A player is liable for injury in a tort action if his conduct is such that is it is either deliberate, willful, or with a reckless disregard for the safety of the other player so as to cause injury to that player. A player is charged with a legal duty to every other player to refrain from conduct proscribed by a safety rule.

When FRAUD vitiates consent o Sexual contact can be the basis of a battery action when you can say that fraud vitiates (negates) consent. o Hogan v. Tavzel: sued for battery after gave her an STD after they had sex. didnt inform her of the STD and didnt take any precautions. If didnt know he had an STD, then probably not liable for battery. However, he DID know. Relevant to his mental state- what he intended when he initiated the sexual contact, therefore its relevant that he knew he had the STD. Fraudulent consent isnt consent. Her consent was fraudulent because she wasnt aware. When a sexual partner conceals that they have an STD, the consent of the other person to sex is vitiated by the concealment.
10

Black-Letter Law: Consent is ineffective if the consenting person was mistaken about the nature and quality of the invasion intended. MUST LOOK AT: The nature and quality of the invasion intended. If the thing you were deceived about were to change the nature and quality of the invasion/contact intended/consented to, then consent is vitiated by the fraud.

Neal v. Neal: cheats on wife, , and does not admit to affair until she confronts him about it. Sues him for battery, claiming she would not have had sexual intercourse with him had she known he was having an affair. The facts were viewed in the light most favorable to her. In doing so, they held that you cant have a battery claim based on your partner committing adultery. Court held that s allegations were insufficient to establish lack of consent. Although deceived as to the exclusivity of their relationship, it did not directly or substantially relate to the essential nature of the physical contact she consented to. Black-Letter Law: Where a persons consent to physical contact is given based upon a substantial mistake, known to or induced by the actor, concerning the nature of the contact itself or the extent of the harm to be expected therefrom, the consent is deemed to be ineffective, and the actor may be held liable as if no consent had been given. Mistake must extend to the essential character of the act itself, rather than to some collateral matter which merely operates as an inducement. The key is whether the misrepresentation is to a central/essential nature of the act (in which case the consent is not valid), or if it is to a material/extraneous element (in which case consent is valid). (Sup Court of Idaho reversed the Court of Appeals decision in favor of her, probably a moral judgment. Decides its closer to essential nature, not collateral, as Court of Appeals had decided.) Hypothetical: What if Mr. A says to Ms. B, have sex with me and I will give you $100. Turns out its a counterfeit bill. He knows its a counterfeit bill. Does the fraud vitiate the consent? She wanted sex + monetary transfer, and she only got sex. Mistake has to lend itself to the character of the act. Collateral matter, not to the essential nature. Hypothetical: Get a root canal; turns out person giving the canal has a disease. Is consent vitiated? Do you let it go to a jury? (Brzoska v. Olsen) Court ruled that there cannot be a battery on these facts unless the battery can show there was an actual channel of disease transmission.

NEGLIGENCE & STANDARD OF CARE Relevance of 19th Century History (of Negligence law) Most modern negligence law developed in the 19th century In order to have liability in tort, you must show that was at fault (Fault-Based) For any kind of unintentional injury, had to show a particular kind of fault (negligence). Negligence (tort) involves the creation of foreseeable and unreasonable risk, usually of physical harm or property damage to another person. Emotional harm, standing alone, will not normally cut it for a negligence claim. Differs from battery law because harm must be quantifiable in terms of physical or property. Why no actual damages in elements of battery? Dignity harms can be compensated by law of battery (ex. Fischer v. Carrousel) but not in negligence.
11

Demands particular kinds of injuries in ways battery doesnt. Great capacity for mass injurers. 19th century: Industrial Revolution, development of railroads/cars, factories & industrial machinery. Relevance to tort law? o More ways people can get injured. o More potential for mass injuries (railroad accidents) o Becomes necessary for liability to be fault-based to protect burgeoning industries. Alternatives to tort system? o Strict liability you hurt someone, you pay. Fault, cautions you took, etc. are irrelevant. o Social insurance system (nationalized health/medical care/workers compensation) o Which system is the conservative option? Tort system (Freedom of action even if you cause injuries, unless you are at fault. Designed to help business ESCAPE tort liability.) Five elements constitute the prima facie case in negligence (meaning the will lose if he fails to establish any): 1.) Duty - The must establish that the had a duty to conform its conduct to a specific standard. a. Only element decided by judge as a matter of law; all others decided by jury. 2.) Breach The must prove that the s conduct failed to conform to the appropriate standard. a. Also known as breach of the s duty, or as substandard conduct on the part of the , or negligence. b. Economics is relevant Were there any cost-efficient alternatives? 3.) Cause in fact must prove that the s substandard conduct was a factual cause or cause in fact of the s injuries. 4.) Proximate cause The must prove that the s substandard conduct was a legal cause of the s injuries. 5.) Damages The must prove actual damages (as opposed to compensatory/nominal/punitive).

BREACH: NEGLIGENCE STANDARDS OF CARE The Reasonable Person Standard o Under this approach, people have a duty to take the same degree of care that a "reasonable person" in the 's situation, with the same knowledge as the , would take under the circumstances = Objective Standard Leading formulation of standard of applicable care in American negligence cases. Grace & Co. v. City of Los Angeles o Pipe-line that ran from a public street into a shed storing 1,960 bags of coffee for the burst, damaging the product. contends the failure of the to regularly inspect the line was negligent. o City always had a do-nothing policy regarding the inspection of the water lines. Inspecting the pipes would be prohibitively expensive and economically unfeasible. The City is required to take only reasonable precautions. o A determination that a failed to exercise reasonable care has 2 components: The fact finder (usually the jury) must determine what the actually did. The fact finder must evaluate the s behavior to determine whether it was reasonable under the circumstances. o Negligence uses a foresight standard, not a hindsight standard. Reasonable care requires only that a person take reasonable precautions with respect to risks that a reasonable person would foresee.
12

Use cost-benefit analysis in everyday life. Make tradeoffs every day and shortchange safety, and thats not unreasonable. City, therefore, probably wasnt unreasonable either.

Hand Formula: United States v. Carroll Towing Co. o Explains three different standards to use when assessing negligence (2nd element) o Contains a famous formulation of the negligence standard called the Hand Formula. Judges usually use this; juries use reasonable person standard. o The duty to take reasonable care is a function of three variables: The probability of an accident occurring (P) The gravity of the resulting injury (L) Look at it BEFORE, what are the worst things that can go wrong? What is foreseeable? The burden of adequate precautions. (B) Utility of Actors Conduct Alternatives? o BPL Formula = B (Burden), P (Probability of something happening), L (magnitude of loss; how bad injury is) Negligent if Burden < Gravity x Probability B < LP o In other words: If the burden of taking precautions is less than the probability of harm multiplied by the gravity of the potential injury, then the defendant has a duty to the plaintiff. o Looks to social value of interests imperiled Extent of Injury Severity Number of people/Property affected o Incommensurability problem no common measuring scheme to determine the quantity of B,P and L. Maybe some way to get the number on P, but the others are unquantifiable. o Another problem information costs need research to determine some of the costs/predictions of benefits and consequences of every action. Tort system encourages actors to ensure the right level of safety. o Another criticism many times an actor has a policy for doing certain things- means they planned it in advance. BPL says when they make this decision they need to consider the burden vs. probability of injury. A lot of negligence has nothing to do with something you PLAN to do, a lot of it is momentary inadvertence. o Another approach to measuring negligence: Foreseeable-danger approach reasonably prudent persons are expected to do everything possible to avoid creating foreseeable dangers to others. o Third approach: Community-expectations approach a reasonably prudent person conforms to the prevailing expectations in the community about consideration for the safety of others. Distinguish between what a jury is typically instructed (to decide if person behaved under reasonable circumstances) vs. what an appellate judge decides (if a jury was correct in deciding a reasonable breach). Does an emergency affect the breach standard? o Yes- In evaluating conduct and deciding if its a breach or not, the law CAN take into account if you were acting in an emergency. Cant be an emergency of your own
13

o o

creation. WANTS emergency instructions to be given, doesnt. Effect of instruction is that it may cause the jury to focus on that facet, which may change outcome. Example: You are rushing someone to hospital and you hit a car, they sue you for negligence. Did you behave as a reasonable person under the circumstances? Is an emergency one of the circumstances you could take into account in deciding if you breached your duty of behaving as a reasonable person? Yes. Consider it as one of the circumstances, but dont change the standard. Jury instructions: give breach instructions but also give instructions that an actor acting in emergency situations is not considered negligent if they had no way to avoid the harm due to the emergency. What difference does it make? Clarifies reasonable standard.

Industry Customs o T.J. Hooper: Coal was lost because barges were in a storm. Owner of coal sued barges under their contract, and barges sued tows, saying they werent reasonably fit to weather a storm. Were the 2 tugs reasonably fit to weather a storm? Appeals court didnt think so Should have had radios. o Industry Custom: not to have receiving radios. Breach determination what was the alternative? BPL Formula used. Is a reasonable person the same as the AVERAGE person?? What the average person does is NOT what the reasonable person does under the circumstances. Emphasis on BURDEN (B) o Compliance with industry customs is some evidence that you didnt breach. By the same token, departure from industry custom is some evidence that you DID breach. However, it is not conclusive. o Mere fact that you were complying does not mean that you did not breach. By the same token, the mere fact that you departed from the industry custom does not mean you did breach. o How do you establish a custom? Expert, building codes, etc. Rationale: people who have thought about it and deal w/ it on a daily basis, and have thought out the cost-benefit and taken the appropriate action. o What is the affect of this decision? Entire industry has to alter their customs because of this judge. o Majority View: Custom is some evidence of the appropriate standard of care but not a conclusive indication of what constitutes due care. Courts may hold the practice of an entire industry to be unreasonably risky. Custom in Medical Malpractice Cases o Locality rule: physicians are held to a standard of care and skill ordinarily possessed by others in the profession in similar localities. o More commonly held view: physicians are held to the standard of care and skill of the average qualified practitioner, taking account of advances in the profession and the medical resources available to the physician. o In general, in medical malpractice cases the standard of care is established by the custom of physicians, in the same specialty, under the same circumstances. o Physicians generally have a duty to disclose the risks of medical treatment to the extent that a reasonable physician would know that a reasonable patient needs the information to make an intelligent choice as to whether to proceed with treatment.

14

The Reasonable Person & Intelligence o Vaughn v. Menlove: , despite warnings, did nothing to prevent his haystack from catching on fire, which destroyed his neighbor, the s, home. Trial court found for , and appealed (rule nisi appeal to an appellate court). claims he should not have been judged for gross negligence by a jury under a reasonably prudent person (objective) standard, but under a subjective standard, judging instead if he had acted bona fide to the best of his judgment, and that he should not be responsible for the misfortune of not possessing the highest order of intelligence. o The subjective standard would be too vague. The objective rule has always been the rule, and a jury has always been able to apply it in cases of negligence. o Law doesnt forgive for lack of common sense; also, then anyone could take up this defense. Rationale for Objective Standard: o Difficult to know/apply subjective standard. o Consistency, uniformity. o Otherwise standard would be too variable. o Discourages fraud & deception. o Know what to expect from other people. o Encourages dumb people to rise to the best of their capacity. (Incentive to learn) EXCEPTIONS: PHYSICAL DISABILITIES o Physically disabled people are held to the standard of care of a reasonable person with the same physical disability or disabilities. o Roberts v. State of Louisiana: was injured after Mike Burson, a blind man who ran the concession stand at the Post Office, bumped into him, causing him to fall. sued State of Louisiana for respondeat superior (vicarious liability) and negligent failure by the State to properly supervise and oversee the safe operation of the concession stand. s expert said its not uncommon for blind people to become familiar with their surroundings and not use a cane. Court concluded Burson was NOT acting unreasonable under the circumstances. Rule with regard to people with disabilities: Need to take reasonable precautions a normal person would take, but can take into account their physical disability. How do you apply rule? WE dont know what a person who is blind does. To figure this out, you need expert testimony. Have to allow inclusion of people with physical disabilities into society, but dont want to make a specific standard for them. Include them in reasonable person standard, but too harsh to hold them to a standard they couldnt meet. Insanity or Mental Disability o Courts will NOT take it into account. Not a circumstance we can consider when deciding whether a person behaved under a reasonable person standard under the circumstances. o How does this differ to physical disability characteristics? Holding people to a standard they cannot meet. o Application: Can see and prove a physical disability (visible), mental instability is invisible (hard to diagnose). Higher information costs of proving mental disabilities. Skepticism/suspicion of the validity of mental disabilities.
15

We preserve rule today, partly because much higher information costs to finding mental disability, and there is more variability to same mental disability, unlike with a physical disability. o One rationale is that people of such low functioning should be kept confined. o In some jurisdictions, insanity may be a defense when: It can be proven that the cannot understand the duty to use ordinary care; The insane cannot exercise due care; or The insane defendant has no forewarning of his/her susceptibility to a mental aberration. Childhood o When a child engages in negligent acts, they should be judged by child standard of like intelligence, age, experience. Subjective components, but objective standard. Blended Standard. With children, you take into account their relative intelligence, as opposed to adults. Maturation process, not the same for every individual. Having experiences leads to maturity, and then become capable of being reasonable people. o Adult Exception Rule: (from Restatement) Children engaged in adult activities should be responsible for their activities, judged by reasonable person standards. Child has to be engaged in an activity in which an adult would be for a child to be liable. Risks are great like driving a car, plane, etc. There are grey areas. Plaintiff v. Child = When child is engaged in adult-like behavior, is entitled to recovery. The child rather than any innocent victims of the childs conduct should suffer the consequences. Child v. Defendant = could have foreseen danger. Adult activity exception doesnt apply. o Strait v. Crary: Strait, 16 at the time, was drinking in Crarys car, 21, and tried to climb out the passenger window into the box of the truck, instead falling and injuring his leg. Strait and his parents sued Crary for negligence in regard to his operation of the truck under the circumstances. At the trial, Strait requested several jury instructions: Wis J I-Civil 1010: Sets a separate standard of care for children. Wis J I-Civil 1582: Instructs jury that it must consider that special standard in comparing the parties negligence. Trial court applied first rule (above), but appellate court says adult exception standard applied because they found there should be a distinction between a case where the child was the defendant and where the child was the plaintiff. When a child is engaged in an allegedly adult activity, the plaintiff is an innocent victim and the plaintiff needs compensation. Court found BOTH of them negligent. If plaintiff is negligent, more so than defendant, case is dismissed. asserts in defense that even though he was negligent, was also negligent in failing to use reasonable care in his own safety and should have an effect on his liability. In common law, it would have had the effect of barring the any recovery AT ALL, even if a tiny bit negligent. Contributory Negligence = failure to use reasonable care for ones own safety. Effective affirmative defense that bars plaintiffs liability. Comparative negligence failure to use reasonable care for ones own safety. Difference from contributory negligence is the EFFECT.
16

If youre plaintiff and youre negligent, but less than the defendant, you can recover percentage. If more, then you can get nothing. Difference from Intentional Torts: Child standard (Garratt v. Dailey) did not apply same standard. IT: Dont cut children any slack unless they are SO young they lack the capacity to form intent. Hypothetical: Common sense that you dont mix pneumonia and bleach together. A child, 12, is very advanced; skipped 4 years of school, genius in chemistry, has a lab set up in garage. Mixes pneumonia and bleach together and causes serious injury to a neighbor. What standard do you apply? If you apply child standard: Child of like age 12, intelligence off the radar, experience very. Chances are hell be held liable. If another kid, same age, not of same intelligence with no experience likely wont be held liable. If its an experienced adult? Do you take into an account that they knew a lot more of chemistry than an average adult? Dont get to catch a break for knowledge you have and didnt use. (Vaughn v. Menlove)

Other special categories under the reasonable person rule: o Beginners are held to the standard of care of a reasonable person of average experience, not the standard of a "reasonable beginner". o Intoxicated persons are held to the standard of care of a reasonable person who is sober. o Poor people are held to the standard of care of a reasonable person, without regard to their indigence. BREACH: NEGLIGENCE PER SE VIOLATION OF STATUTE Negligence per se = negligence in itself Evidence that the has violated a statute is conclusive evidence that has breached a duty of care toward the if: o The was among the class of persons the statute was enacted to protect; and o The s injury is the kind of harm the statute was enacted to prevent. Questions to ask when dealing with statutes: o Remember that the injury is seeking to discern the legislative intent, or purpose, underlying the statute. o What kind of statute is it? Is it a safety statute? If not, it may not be relevant at all. o Who passed the statute? Federal? State? Municipality? o What class of persons is the statute designed to protect? o What type of harm is the statute designed to prevent? o What procedural effect should be given to violation? Negligence per se/irrebuttable presumption of breach? Or something else (mere evidence of negligence)? o What are the possible excuses? Martin v. Herzog o and husband were driving on a road at night in their buggy when an oncoming car hit them, killing the husband. brought suit against driver for negligence for not keeping to the right of the center of the highway. Negligence was also brought against s
17

o o

o o

intestate for traveling without lights, which is a statute. Trial court found delinquent, and Appellate Division reversed and ordered a new trial. Unexcused omission of statutory signals is evidence of negligence on the s part. Violation of a statute is negligence. In this case, there was an instance of the admitted violation of a statute intended for the protection of travelers on the highway. The omission of lights was a negligent wrong. More importantly, the court found that it was CONTRIBUTORY negligence, finding a causal connection between negligence and the injury. The collision was due to his failure to see at a time when sight should have been aroused and guided by the statutory warnings. A statute designed for the protection of human life is not to be brushed aside as a form of words. Appellate court says s conduct constituted negligence per se: BREACH, other elements of tort still need to be made out. If you show that violated statute, you have made out breach. The statute in this case was a safety-protecting statute: As part of duty, you must conform to safety protecting statutes in your jurisdiction. Failure to do so is negligence in and of itself.

Rationale for why in tort case you borrow statute: o Comity (Reasonable person is general standard; legislative standard is specificsets what a reasonable person would do.) Cant default to the general floor when the legislature has set the floor more specifically above the floor of reasonable person under the circumstances. Deference to the legislative-presumed intent /judgment how reasonable people behave. o Reasonable people typically adhere to criminal statutes that have purposes of preserving safety. Ordinance vs. Statute o Ordinance is local, usually passed by county, city, and municipality. Restricted sphere. o Statute is passed by legislature for entire state. o Cardozo says an ordinance is still a law that sets safety standards. o Violation of ordinance usually treated as some evidence of negligence (not sufficient to establish, but jury does hear about it). o In contrast, a violation of statute is treated as negligence per se = inquiry over. Breach is established. Hypo: statute says taillights need to be in working order. Both of As taillights are out, in violation of statute. o Someone crosses center lane and runs into As car. o If somebody had rear-ended him in a collision, he being in violation of statute would be negligent per se. o However, he was going forward and person hit from front, taillights had NO BEARING. NOT A CAUSE therefore violation of statute doesnt apply. o Cardozo authority for saying this: We must be on our guard against confusing the question of negligence with that of the CAUSAL CONNECTION between the negligence and the injury. Hypo: has a buggy and has not paid the excise tax for the buggy (revenue-raising tax). Same facts as Herzog. Is the buggy negligent?
18

o o o

Heart of inquiry is what you think legislatures purpose was and whether you think the purpose was safety-promoting. If purpose of tax was meant to keep cars off the road, may be contributorily negligent. If it was to raise revenue, maybe not. **Sometimes have to infer purpose of statute. If statute is not about SAFETY, do not apply it.**

Hypo: A statute says you may not sell cigarettes to children. A person sells to a child, and there is clear evidence its not a strict liability statute. Child then sets a fire with cigarette and causes $100,000 of property damage. How do you know what questions to ask? o A rule less rigid has been applied where the one who complains of the omission is not a member of the class for whose protection the safeguard is designed.- Cardozo o Are the people who were injured (property owners) a member of the class for whose protection the safeguard was designed? o 1 argument with cigarette statute is that statute seems like it was to protect children from the harms of smoking, not to protect property owners, therefore statute is irrelevant to breach determination. o RULE: If statute doesnt apply, default back to reasonable person standard!!!! o HOWEVER: Could argue that the statute is not just to protect children from smoking but has a secondary purpose of preventing children from mishandling cigarettes in a way that could damage property. COMPLIANCE WITH A STATUTE o Usually evidence of reasonable care, but not definite. Shows you were following the rules, but maybe those particular rules should not have been followed in this particular case. o Hypo: An ordinance requires that all trains coming into the city have whistles with 100 decibels. Train comes through with 100 decibels, therefore not negligent by not warning the car when it got hit by the train. Evidence shows train was going right by a waterfall, so 100 decibels may not be enough. Negligent, even though you followed statute? Yes, because sometimes you have a duty to go above what the statute says. Reasonable person would go above statute to tailor it to the circumstances of the case. o Tedla v. Ellman: s were on the right side of the eastbound road, pushing a baby carriage. Very heavy traffic going westbound. No traffic going eastbound. Statute provided that pedestrians should walk facing the traffic. Plaintiffs were walking WITH traffic, should have been on westbound side traveling toward traffic- clearly in violation of statute. Struck by a car and she was injured & her brother was killed. By observing the statute under these circumstances, the danger would have been greater. Certain excuses justify for not treating a violation of statute as negligent per se. RULE: **If compliance would have created a greater danger, it constitutes as an exception to the negligence per se effect.** Other Exceptions: Incapacity, lack of knowledge needed to comply, inability to comply, and emergency. Exceptional Statutes (irrebuttable presumption- strict liability!)
19

o o o

Strict Liability: meant to protect a class of persons who are unable to protect themselves (i.e. minors, intoxicated persons, etc.); safety statutes. Violation of these statutes is usually a lower penalty (misdemeanor). Must determine the legislative purpose behind the statute. Doesnt matter if intended to be guilty absolute liability. Zerby v. Warren: Zerby (14) and Reiken (13) bought glue from Dieke, from a store owned by Warren. Zerby died from inhaling glue. His family brought charges against s, who brought suit against Reiken seeking compensation, and also a suit against U.S. Plywood for indemnity, excusing all liability. Zerbys family then amended their complaint to assert claims against Reiken and U.S. Plywood. All s pleaded affirmative defenses of contributory negligence. Trial Judge made pre-trial ruling that Minn. Stat. 145.38 imposed ABSOLUTE liability on Warren & Dieke, because of exceptional nature of statute, and could not use contributory negligence as a defense. Statute said: No person shall sell to a person under 19 years of age any glue or cement containing toluene etc. Second part says no person under 19 shall use or possess glue of substance containing toluene with intent of inducing intoxication Doesnt say its a criminal statute and doesnt specify strict liability. So how do you determine? Statute is VERY specific. Kids people under 19 (minors). Minors have been known to sniff glue statute establishes a public policy to prevent this. What is difference between 1st and 2nd part of statute? 2nd helps support the policy of the 1st provision. Dont want minors possessing the glue, but would violate 1st part of policy if we let you get off the hook for the bad behavior of the minor.

3 Procedural Effects: (depending on jurisdiction) o Negligence per se with excuse Irrebuttable Presumption Meaning any party who violated the statute may offer evidence of a recognized excuse or justification for violating it. Deemed negligent without evidence of an excuse. 5 listed exceptions under Restatement; although list not exclusive. o Incapacity o Impossible to comply o neither knows nor should know of reason for compliance o Emergency o Compliance creates greater risk than violation (Tedla v. Ellman) If presents evidence, s effort to prove a violation of the statute would fail. Jury would then be instructed to determine if acted reasonably under the circumstances. Burden remains on plaintiff! o Presumption of Negligence Proof of a statutory violation creates a presumption that violator was negligent. Violator still free to rebut the presumption by showing that the reasonable person would have acted as he did. If violator offers evidence of a good reason for his conduct, jury is left to asses conduct under reasonable person standard; if not, violation of statute establishes negligence. Difference between this & negligence per se with excuse : There is no specific list of valid excuses under the presumption approach.
20

free to offer any competent evidence of an appropriate reason for violating the statute. Burden remains on plaintiff! In both presumption & excuse approach, if violated statute & offers no excuse, a judge should instruct the jury that they must find the negligent. Evidence of negligence (minority approach) Evidence that violated a statute is admissible at trial; jury may consider it along with other evidence that did or did not exercise ordinary care. They may be persuaded that was negligent, but they are not compelled to find him negligent, even in the absence of rebutting evidence from the . Proof of an unexcused violation would support a finding of negligence by the jury, but they would be free to find the was not negligent, even if no excuse was offered.

BREACH: CIRCUMSTANTIAL EVIDENCE / RES IPSA LOQUITUR (The Thing Speaks for Itself) Res Ipsa Loquitur (RIL): Doctrine developed to deal with problems relating to circumstantial evidence. o Circumstantial evidence is not inevitably weaker than direct evidence. When o o o would direct evidence be weaker? If it doesnt really point in one direction. Eyewitness testimony can be defective. Very unreliable.

Res Ipsa is merely one type of circumstantial evidence. It is used when: o (1) The event is of a kind that ordinarily does not occur in the absence of someones negligence; o (2) It is caused by an agency or instrumentality within the exclusive control of the ; (purpose of this requirement: bringing the negligence home to the ; it is not enough to show someone was negligent; it must be more likely than not it was the who was negligent); and o (3) It must not have been due to any voluntary action or contribution on the part of the . This requirement has been softened due to some courts allowing comparative negligence. Often solves a proof problem. o Function: doctrine that either forgives deficits in proof that would otherwise be required; comes in to help s solve proof problems. Trick is to know WHEN this applies. Doctrine comes from British case Byrne v. Boalde o has 2-story flour warehouse. In loft of warehouse are s employees, messing around with flour barrels. o driving outside when all of a sudden a barrel of flour falls out of warehouse and hits . o Likely problem in pinning negligence on someone: no one around to see it happen, hard to assign direct proof. Has to rely on testimony to get any evidence. Problem with that: people in loft claim not to have seen anything.
21

o o o o

Obvious to court that barrels of flour shouldnt be falling out of warehouses if reasonable care is taken. Public policy: is it fair to help ? Is it fair to run the risk of making pay if he wasnt negligent? Most likely scenario: Employees of warehouse probably werent paying attention, and must have been negligent in some way for allowing the barrel to fall. Doesnt matter that we cant pin it on one employee because of VICARIOUS LIABILITY.

Significance: o (1) Usually, the doctrine is invoked by the at trial to defeat a motion by the for directed verdict following the close of the s case. o (2) Most courts which rely on res ipsa loquitur consider the s circumstantial evidence to be prima facie evidence that the has been negligent. o (3) In a small number of cases where res ipsa loquitur is invoked, the s circumstantial evidence is considered strong enough for the court to direct a verdict for the following the close of the 's case. Procedural Effects of RIL: o Still have to prove 4 other elements of negligence. o (1) Permissive Inference of Breach: Most common treatment of RIL. Jury can think about it, and can decide that breached or did not. PERMITS a jury to infer breach from the circumstances. o (2) Rebuttable Presumption of Breach: Stronger effect/standard. BURSTING BUBBLE PRESUMPTION. Anytime an element of a cause of action (like breach element), usually has 2 burdens (production & persuasion; must produce evidence & persuade jury that more likely than not breached). This shifts burden from the to the to prove he did not breach breach is established unless he says something to the contrary (rebuttable presumption). Burden of production to ; has to come forward with some evidence or he loses on breach issue. o If he satisfies burden of production, then loses, because burden of persuasion still lies on the . Burden of persuasion Must show more likely than not that the breached. o If its 50/50, and meets burden of production, loses. Bursting Bubble Presumption: says he is not negligent and provides reasons/evidence. Once he comes forward with any evidence, balloon bursts and burden of proof stays on the ; still has to convince jury more likely than not the was in breach. If bubble isnt burst, it is deemed to be persuaded. Judge probably should direct the verdict on the breach issue only. If jury finds RIL factors are present, then breach is established (unless bubble is burst). o What difference does it make? RIL helps get foot in the door otherwise jury will dismiss case & say there is no breach.
22

Doesnt necessarily ESTABLISH breach; not as good as negligence per se; procedural effect is less. Negligence per se (as in statutory standards) = irrebuttable presumption of negligence (or breach) means that breach is made out, period, UNLESS by a recognized excuse.

Colmenares Vivas v. Sun Alliance Insurance Co. o s riding on escalator in Puerto Rican airport, railing moving up but stops and stairs keep going, wife falls, husband tries to help her and also falls. Sue Puerto Rico Port Authority because they control the airport premises. Also, Westinghouse sued, who is the company in charge of the maintenance and repair of the escalator. This responsibility was delegated to them by Port Authority. o 1.) Is this the kind of accident that occurs unless someone has been negligent? Court held that this was met because an escalator handrail probably would not stop suddenly while the escalator continues moving unless someone had been negligent. o 2.) Argued that they had delegated the duty of maintenance to this company. Port Authority said theyre not in exclusive control because they hired Westinghouse to maintain the escalator. Court ruled that it was a NONDELEGABLE duty. Even if contracted out, the burden and responsibility of it still falls on your shoulders because it is a public facility of high traffic where safety is so important. Impose extra burden and care. How do you know a nondelegable duty when you see one? When responsibility is so important to community that it cant be transferred to someone else. Could be determined by a statute, or when its an issue of safety or public policy, court tells us when we have one. What is the point of an exclusive control requirement? Instincts have to tell us who the is; the most likely person. And instincts have to tell us its most likely negligence. o 3.) Was the negligence due to any voluntary action on the part of the ? doesnt have to totally exclude themselves from the harm; courts are forgiving and they can still recover comparative negligence As opposed to contributory negligence where s would be barred from recovery if they were shown to have been negligent at all. Court in this case found that there was no evidence that the s caused the accident. o All 3 requirements met, and should be allowed to go to jury to determine if Port Authority was liable based on the inference of negligence raised by the application of res ipsa loquitur. Comparative vs. Contributory Negligence o Plaintiff failed to use reasonable care for own safety. Effect is different. o Contributory Negligence: At common law (early common law; Martin v. Herzog): If was contributorily negligent, gets ZERO. o Comparative Negligence: s recovery is reduced proportionally to his negligence. If was 40% negligent, then gets 60% of the damages. PROXIMATE CAUSE UMBRELLA TERM Must be 2 essential elements of proximate cause: (umbrella term)
23

o o

Cause in Fact: Factual causation Foreseeability: Legal causation (proximate cause)

CAUSE IN FACT FACTUAL CAUSATION Required in intentional tort and strict liability cases. o One of elements of the tort of battery is that must have caused physical contact. o In all tort cases, must prove by a preponderance of the evidence (more probable than not) that the s substandard conduct was a cause of the harm or result complained of. o Law does not require to prove that s conduct was the cause of the harm, just A cause. Cause-in-fact requirement is satisfied if s conduct is shown more probably than not to have been among the causes of the result the seeks to attribute to the . Bedrock requirement of the law. o The has the burden of proof as to causation in fact. But-For Test o But for the s conduct would the have been harmed? o If no: was a cause in fact of s harm. o Most cases decided by this test; only a few decided by substitutes. o Make connection between breach & damages cause in fact provides link. But for the s breach of duty of reasonable care (IDENTIFY WITH SPECIFICTY), would the had suffered damages alleged in this case? Example: But for the failure to remove the rowdy people, would the have been struck by the whiskey bottle and injured? (East Texas Theatres, Inc. v. Rutledge) o Sine qua non: without which not test. Indispensable element. o Negligent behavior that does not cause harm usually does not create any liability. o Test asks you to apply a counter-factual hypothesis: once you identify breach with specificity, pretend the party had not performed the alleged negligent action. Would the same injury still have happened? Factual or actual causation. How do you know what would have happened? You dont. Send to jury to determine if its at all arguable. o Every BUT-FOR cause gets you past CIF hurdle. o If not in our realm of common sense, experts could help testify causal links in but-for test. o Statistics RULE: Mere statistical likelihood/probability standing alone without something more is not enough to get past hurdle of but-for. Example: Small town of 20,000 people; only one bus company, X, with blue buses. walks across street and hit by blue bus. Cant testify for sure if its company X that was liable. All evidence of causation you have is that there is one bus company in town and its buses are blue. Has established enough to get past dismissal of the complaint or summary judgment or directed verdict? Enough to get to jury? Probably not. A bus from another town could have come through. However, most likely scenario is that she was hit by a bus operated by X. Must show they had bus routes at that time and place where was hit. Contrary to Rule: Kaminsky v. Hertz Corp held that showing that Hertz owned 90% of the yellow trucks bearing the Hertz logo sufficed to establish that Hertz probably owned the otherwise unidentified one that injured them. This is a stretch of the rule, but you normally need percentage +.
24

Courts are suspicious of statistical evidence; they want it to be more individualized; want to do a little more work to find individualized information to find responsible. Judge ONLY determines whether has come forward with enough evidence to show there is enough breach of duty to act. If he finds that, it goes to jury to determine. These must all be shown by a preponderance of the evidence.

Limited Purpose Substitutes for But-For Test: Substantial Factor Test o Was the 's conduct a substantial factor giving rise to the 's harm? If yes: was cause in fact of 's harm. This is a minority approach. o If two forces are actively operating, one because of the actors negligence, the other not because of any misconduct on his part, and each of itself is sufficient to bring about harm to another, the actors negligence may be found to be a SUBSTANTIAL FACTOR in bringing it about. Restatement (Third) of Torts: When an actors tortious conduct is not a factual cause of physical harm under the [but-for test] only because another causal set exists that is also sufficient to cause the physical harm at the same time, the actors tortuous conduct is a factual cause of the harm. o Apply when you have 2 factors when each acting independently would have caused the harm. with reasons beyond their control could not say with certainty which factor caused the harm. Has to be more than one tortfeasor & cant determine which caused the injury. Ordinarily when but-for fails loses; on burden of to find with a preponderance of the evidence that caused the harm. This is a scenario where we forgive for not being able to pinpoint the . o Basko v. Sterling Drug, Co. develops skin disease. Took Aralen 1953-1957. Took Triquin 1959-1961.Took Atabrine, but didnt have the right chemical properties to cause the skin disease leading to blindness, so its irrelevant in this case. has a proof problem- dont know which drug caused it; both have same chemicals in them- chloroquine. Chloroquine was known to cause these side effects in 1959, which means that after 1959 someone was negligent in failing to warn of the risks. only had a duty for after 1959. Aralen- may not have been negligent. If no breach- off the hook. Triquin could have been negligent because she took it from 59 to 61. Did Triquin cause her blindness? But for the manufacturer of Triquins negligent failure to warn of the blindness side effect, would the had ended up blind? She had ALREADY been taking Aralen, so taking Aralen standing alone may have ultimately lead her to be blind even if she had never taken Triquin. Should the manufacturers of Triquin be off the hook for being negligent just because she had taken Aralen before? Essence of breach- Risk Creation. Usually you are liable when your risk creation causes someone harm. Is it Aralen+ Triquin that produced the blindness? If so, does but for test work? No. If only took one drug, but-for test works.
25

Court says either of them alone could have caused it. Should s be off the hook if but-for doesnt work??? Idea of Substantial Factor Test: we know Triquin materially contributed in some way to s harm. Not going to let off the hook just because someone else screwed up too. What policies in Basko case support keeping Triquin on the hook? the single tortfeasor cannot be allowed to escape through the meshes of a logical net. He is a wrongdoer; let him unravel the casuistries resulting from his wrong.- Learned Hand Rationale for test: has committed a wrong & this has been a cause of the injury. Such negligent conduct will be more effectively deterred by imposing liability than by giving the wrongdoer a windfall in cases where an all-sufficient innocent cause happens to concur with his wrong in producing the harm. Innocent + a wrongdoer who needs deterrence; shouldnt get off on a technicality. Merging Fires Hypothetical: 2 fires, A & B, operating at same time. Both come across a house and burn down the house. Does the but-for test work? No. But A & B cant be let off hook because both were negligently set. Dont want to pin the harm on an innocent cause. Dont want to let two negligent tortfeasors off. Both engaged in risk creation, particularly when you have a who needs compensation = public policy.

Limited Purpose Substitutes for But-For Test: Alternative Liability o When to apply: Both are tortfeasors; were negligent in exactly the same way. Harm caused by only one Where the conduct of two or more actors is tortious, and it is proved that harm has been caused to the by only one of them, but there is uncertainty as to which one has caused it, the burden is upon each actor to prove that he has not caused the harm. Negligence of s deprives of proving CIF: proof problem. o Seems fair to get away from but-for test under the unique circumstances of the case. o Pennfield Corp. v. Meadow Valley Electric, Inc. Pigs died due to defective electrical ventilation system. MVE joins York and TriState Electric. Both supplied cables. Proof problem- doesnt know which company supplied the faulty cables. One arguably had defective cable & one didnt. Court didnt allow alternative liability in this case because MVE acknowledged that only one actor MAY be tortious while admitting the other may not be. The burden of proof remains on MVE. But-For test doesnt work. Use alternative liability theory: cant figure out which one is liable, so shift burden of proof onto s to sort out which is negligent. Summers v. Tice: 3 guys hunting, 1 is uphill than other 2, gets shot in eye. Both hunters below shot at same time. Which hunter did it? will fail unless he can pin liability on one of them. Court utilizes alternative liability: Where the conduct of two or more actors is tortious, and it is proved that harm has been cause to the plaintiff by only one of them, but there is uncertainty as to which one has caused it, the burden is upon each actor to prove that he has not caused the harm.
26

o o

Likely result: collect from the one with deeper pockets. Hold them jointly & severally liable you can choose how you collect. Joint & several liability: protect the from the risk of not being able to collect, for example, if one tortfeasor is insolvent and the other has the money to pay the damages, the victim can collect the entire amount from the one that is able to pay. Deep Pocket Problem: can seek compensation from only 1 if he chooses, and 1 can ask 2 for contribution, if 2 is solvent. If is put on hook for entire thing, they have a right to ask the other for a portioncontribution. Tort reform goal: to abolish joint & several liability; limits s ability to collect. In Pennfield, cant say for sure which party was negligent. Distinguishable from Summers because you run the risk in Pennfield of pinning the liability on a tortfeasor who never breached. In Summers, you have innocent party, and definitely one wrongdoer. In Summers, there is a 50/50 chance the shooter was one or the other. Is it fair to keep both on the hook for damages? Justification for applying alternative liability: s are better placed to explain what happened than did. Also, wrongdoing s & innocent - cant prove causation under but-for test because there are multiple tortfeasors. Negligence of s deprives ability to prove causation. About individual responsibility, must show culpability & show causation of breach. In most cases in which the alternative liability theory has been used against more than two tortfeasors, there was arguably an element of concerted action that might have provided a better justification for treating each of the tortfeasors conduct as a cause in fact of the harm.

DES CASES: MULTIPLE TORTFEASORS o Social problem: wont miscarry if you take this drug; caused cancer & other serious problems in daughters of recipients. o Involves a signature injury, which helps establish cause in fact. o Problem isnt if DES caused harm, but which manufacturer produced the specific one that the plaintiffs mother took. Hard because isnt the one that took the drug. Difficult to prove. o Factors of DES Cases: Drugs of all companies have same chemical properties; same harm-causing properties. All companies relied on same set of data; no one did independent studies. FDA immune from liability because they dont have the resources to test every product. Latent effects of drugs make it hard to solve proof problem. Each drug behave in the same way: hard to pinpoint specific s Shouldnt be a basis of denying recovery to an innocent . o Flipside: If we deny s recovery, sets no incentives to manufacturers of drugs. o Statute of Limitations problem: Problem is many injuries dont manifest until you are 25+. Had to ask legislature to open up a whole body of tort suits that otherwise couldnt be brought tort expansion, not tort reform. Must convince legislatures its a terrible injustice done to many people. o Next, must prove breach- drug companies acted unreasonably.
27

Causation no clear cut substitute that would solve DES case. Which substitute would work better?

CONCERTED ACTION o Have to prove their actions encouraged similar actions by other companies. o Have to do it w/ an understanding with other companies; did it all together. Like conspiracy theory. o Agreement to engage in this behavior together caused injury. Meeting of the minds caused result = fair to hold both liable. Example: Drag racing down 34th street both are culpable for any harm done. Distinguishable: Both driving down 34th, speeding, half a block apart & other people are speeding too. One person runs over . Dont hold others liable for also speeding. Why is this different? Didnt have an understanding with each other. Drag racing case there is an understanding you will engage in this conduct together. Its parallel but theres no even implicit meeting of the minds. o Mere knowledge by each party of what the other is doing is not sufficient concert to make each liable for the acts of the other since one man ordinarily owes no duty to take affirmative steps to interfere with anothers activities absent some special relationship. It is essential that each particular defendant who is to be charged with responsibility shall be proceeding tortiously , that is to say with INTENT to commit a tort, or with negligence. o Bichler v. Eli Lilly & Co. Mother took DES and daughter developed cancer. Sued Lilly (biggest company involved) under theory of concerted action. Concerted action: all those who, in pursuance of a common plan or design to commit a tortious act, actively take part in it, or further it by cooperation or request, or who lend aid or encouragement to the wrongdoer, or ratify and adopt his acts done for their benefit, are equally liable with him. Lilly could argue that they all happened to do it the same way to make profit, doesnt mean they did it together with any understanding- all competitors. Usually parallel action isnt enough to make it concerted action. Lilly argued that they needed a PLUS-FACTOR: said they were engaged in parallel activities, had a plus factor, and cant be concerted action. Court says too bad. Rationale: concerned action means that persons acting independently of each other in committing the same wrongful act, but although acting independently, their acts have the effect of substantially encouraging or assisting the wrongful conduct of the other, which, in this case, was the alleged failure to adequately test. Rule: CONSCIOUS parallel conduct by itself is enough to form the basis of concerted action. Agreement can be inferred from parallel activity if it manifests an understanding. As a result of concerted action court holds Lilly accountable for ALL of the DES cases- 100% of the damages. Implication of applying this joint & several liability. Very pro- way of solving causation problem- makes deep pocket responsible for all of the injuries. Not the most common approach. Arguably fair because 1 is a wrongdoer- and they agreed to harm.
28

Alternative Liability vs. Concerted Action o Concerted action must be implicit agreement to engage in this activity; cant be merely parallel activity If you find concerted action, the nature of the causation problem then changes to the agreement that caused the harm- then you apply joint & several liability dont shift burden of proof. o In Summers v. Tice, in contrast, if A shows he didnt pull trigger and B did, you can get off the hook. o There is no getting yourself off the hook in concerted action. o Only option you might have it for asking for contribution from fellow drug manufacturers, even then there is no right to compensation.

Enterprise Liability o Example: Very small number of blasting cap manufacturers in the United States. If you were going to buy one, going to get from one of the 6. Someone was injured by one but couldnt tell which company it came from. Enterprise solves causation problem by saying this is a very concentrated industry, covers the entire national market, its less likely to hold someone liable that isnt. Entire concentrated industry ends up being held liable because theyre in a trade association together, which develops standards of how to make blasting caps they all got together and agreed how to make them. They all pay equally, really an extension of concerted actionagreement & delegation to the trade association. Apply joint & several and let them split it among themselves. Why not apply this to DES cases? Too much variability, also very decentralized industry. Market Share Liability o Only applies (in some jurisdictions) where many manufacturers produced an identical (i.e. fungible) defective product, only one manufacturer's specific product caused the harm, and the manufacturer of that specific product cannot be identified. o Hymowitz v. Eli Lilly &Co. Court said concerted action theory does not fit multipleproducts problem. DES companies were engaged in extensive parallel conduct in developing & marketing DES. There is nothing in the record beyond this similar conduct to show any agreement, tacit or otherwise, to market DES for pregnancy use without taking proper steps to ensure the drugs safety. PARALLEL ACTIVITY, WITHOUT MORE, IS INSUFFICIENT TO ESTABLISH THE AGREEMENT ELEMENT NECESSARY TO MAINTAIN A CONCERTED ACTION CLAIM. o must join as s a "substantial percentage" of manufacturers of the generic product. o Generally, the burden of proof as to causation in fact shifts to the s - i.e. in most "market share" states, each has the burden of showing that his or her product did not cause s harm. Each who cannot meet that burden of proof is liable for the proportion of the judgment represented by that s share of the market, i.e. the percentage of the entire production of the defective product that the s product represents.
29

o o o

Some states hold the companies liable regardless if they can prove their product did not cause the s harm. Example- 1 , 5 s manufactured DES, 1 had 40%, 2 had 10%, 3 had 5%, 4 had 30%, and 5 had 15%. doesnt know whose DES mother took, sues all 5 in market at time; all negligent in approval and marketing of DES. If damages are $100,000, should be able to get 40,000 from 1, 10,000 from 2, etc. collect proportionate share of damages based on market-share percentage. How is this fair? If s are always liable in proportion it should equal out (law of large numbers) if you pay out every in this way. Market share is a theory of joint causation. Aggregate solution. Apply -SEVERAL liability, not JOINT & several: can NEVER be responsible for more than their share. If is insolvent, too bad for = has incentive to include as many manufacturers as possible. s prefer concerted action over market share liability because of deep pockets; they want joint & several.

Limited Purpose Substitutes: The Lost Opportunity Doctrine o Under the traditional view, where the medical malpractice of a health care provider reduces a patient's chances of survival, the in a wrongful death action must prove that but for the 's negligence the 's decedent/patient would probably have had at least a 50% chance of survival. However, a number of states have taken the view that, in these cases, the must only prove that the 's malpractice deprived the decedent of a "significant chance" to survive or recover, even if, but for the 's malpractice, the decedent would probably have had less than a 50% chance of survival. o Grant v. American National Red Cross charged with not testing ALT, which COULD link to hepatitis C. At BEST estimate, only 40% of the cases would have detected hepatitis C. received transfusion and contracted hepatitis. Causation problem: must prove BUT FOR failure to test donors blood for ALT, Grant would not have contracted hepatitis C. CANNOT establish this. MOST of the time test would not have found this. Court found that failed to prove with a high likelihood that screening the blood for ALT would have offered more than a 30%+ chance of detecting a donors hepatitis. In contrast: Negligent misdiagnosis of cancer, ends up dead. Expert testifies if cancer had been diagnosed when first went to doctor, would have had a 90% chance of survival. Does have causation problem in proving that negligence of doctor caused the to die of cancer? Dont know if he would be in the 90% or the 10%; could give him 90% of the damages. Loss of chance case. Difference between this & Grant: greater than 50% likelihood that the would have lived. o Burden of proof on causation - have to prove more likely than not that the negligent misdiagnosis caused to die, doesnt seem that hard if greater than 50%. Doesnt have to be THE cause, just A cause.

30

Argument: lost chance for better outcome. Given difficulties of quantifying this, courts dont bend normal causation to stretch to cases where you just lost a better outcome as opposed to chances of SURVIVAL. Herskovits v. Group Health Co. established that damages would be allowed when the medical testimony put the chance of survival that the s negligent conduct cost the patient at less than 50%. Became the leading case for the lost opportunity doctrine. Negligent failure to timely diagnose & begin treating the patients lung cancer lowered the patients chances of surviving the cancer from 39% to 25%. Court held that damages should be awarded (14% of what the damages would have been if had been able to prove s negligent conduct caused the death.) The arguments for using the lost opportunity doctrine in the medical malpractice context are very strong. Damages for loss of a chance are necessary to prevent the underdeterrance of medical negligence. Doctrine is controversial: majority of jurisdictions have rejected it. They may have been one of the people who couldve survived if under 49% if they had been diagnosed early enough deprives them of their lives, and something they consider to be very valuable. Lost opportunity doctrine has almost universally been confined to medical malpractice cases because: A contractual relationship exists between patient & physician, in which the nature of the contract is that the physician will take every reasonable measure to obtain an optimal outcome for the patient; Reasonably good empirical evidence is available about the general statistical probability of the lost opportunity; and Frequently the consequences of the physicians negligence will deprive the patient of a less-than-50% chance for recovery. Negligence caused death, deprived person of chance of survival injury Example: Person has 40% chance of survival, because of delay in diagnosis they now have a 5%, and then they die. (35% reduction of survival) But for negligence, would person have been deprived chance of survival? (Reconceptualize the injury- then causation problem becomes a damages problem.) 3 ways to calculate damages if shows loss of chance of 50% or less: (1) Proportional Recovery: Most common approach give percentage of damages that they lost. Fractional recovery based on percentage chance you were deprived of. Example: If has 90% chance of survival, due to misdiagnosis now have 0%. (Met their burden, deserve full compensation). o Damages to are $1 million. would get all of it. How much would they get if only 30% survival and court allows proportional recovery? o $300,000. Criticism: Either over compensates, or under compensates if you were one of the lucky people who would have lived. Dollar figure bears no resemblance to what really happened. (2) Give 100% if jury wants to Because of criticism under the first approach, some courts allow jury to decide if they would have lived anyway, and then they would get 100% recovery.
31

(3) Let jury put a number on all the circumstances Instructs jury to consider all the evidence, including the testimony assigning numerical values to the plaintiffs chances, and come up with its own valuation. Tell jury they had 30% chance of survival, let jury decide damages. Increased risk theory: s can recover if the negligence created a small chance of future risk; minority viewpoint.

Apportioning Harm to Multiple Tortfeasors: Joint & Several Liability o 3 main bases for application of J&S: 1-Concerted action (drag racing) 2-Indivisible injury (Piner case) 3-Application of vicarious liability o Indivisible Injury In a normal case, the application of J&S liability doesnt offend ordinary principles of causation; but-for test works- just a mechanism for collecting damages. Hypothetical: Car B is speeding & car A runs stop sign; collide at intersection & passenger C is injured. But for the negligence of A in running the stop sign, the accident wouldnt have occurred. But for the negligence of car B in speeding, the accident would not have occurred. But-for test applies to both s; each is a 100% cause of the accident; a but-for cause. How do you apportion damages? Typical indivisible injury case. No causation problem- apply J&S liability to appropriate damages. Effect of this: allows to collect 100% of damages from either she chooses, or split it as she chooses. If A is the deep pocket, C can sue A and get 100% of the damages. Usually, you add to the J&S liability doctrine a right of contribution, meaning A can then collect from B. Allows to collect from , and to collect from other s. o Divisible Injury Hypothetical: Shooter A shoots in arm, shooter B shoots in other arm. A is responsible for 100% of wound A and B is responsible for 100% of wound B. Typical divisible injury case: no basis for application of J&S. Hypothetical: B shoots in arm; goes to hospital and doctor commits malpractice in treating that wound. already had broken arm; & now arm amputated because of doctors negligence. Divisible injury between A & B; but indivisible injury between B & doctor. In first hypothetical: Doctor also liable for injuries exacerbated; medical negligence is considered foreseeable; on the hook together: J&S liability. However, if doctors negligence was GROSS, then doctor is alone liable for his injuries; A & B only liable for injuries THEY caused, so J&S is not applied. o Piner v. Superior Court- was in successive collisions on same day; doctor didnt examine him in between collisions; same injuries from both and couldnt attribute to which accident injuries came from. Theoretically divisible because if dr. had checked him out before crash the injuries could have been divided; but he didnt, so the injuries are practically indivisible.
32

says treat this as an indivisible injury and let s apportion it amongst themselves. If they cant, let jury hold them EACH liable for a portion of it J&S liability. Why is a candidate for bending the normal rules of causation? Multiple culpable actors. NORMAL RULE: apportions damages between multiple tortfeasors; but no way to apply this- 2 wrongdoers would get off the hook, so its just in this scenario to keep them on the hook and let them or jury figure it out. UCATA: Uniform Contribution Among Tortfeasors Act (Background) Step 1: (Early Common Law) NO CONTRIBUTION AMONG TORTFEASORS If youre going to apply J&S liability, you can pick which & thats the end of the story. o Idea among torts is that if youre a wrongdoer & your actions caused 100% of the injuries, injuries arent going to be sorted out when you have unclean hands. Damages dont have to be adjusted. o Other effect: In early common law, if was involved in any wrongdoing at all, they got nothing. (Martin v. Herzog). Dont adjust things between wrongdoers in early common law. Contributory negligence, no comparative negligence. Step 2: Shift in mentality. If theyre all wrongdoers, deterrence would require that everyone pay. Still dont think we can apportion things that well; so apply contribution on a per capita basis if you have 2 s, each pays half. 3- each pays 1/3, etc. Step 3: Shift again once comparative fault is adopted- this is about allocation between and ; also allows contribution, which is allocation between 1 & 2. o Comparative fault between & - if we can say was 30% at fault and was 70% at fault, why cant same thing be done among s? o Contribution should be done on a percentage-fault type basis. o Shift to comparative: assigns a percentage to the and diminishes recovery to . Allows contribution between tortfeasors on basis of percentage fault. o Example: If is 30% at fault, and s A & B are each 35% at fault, and has $100,000 in damages, can collect max $70,000. If collects $70,000 from A, A can get $35,000 from B. Summary: *1984 version: shift from contributory comparative fault; allowed contribution between s on percentage-share basis. o [Note: at early common law, there was no contribution; at intermediate stage, there was contribution based per capita; once comparative negligence is adopted, there is contribution based on percentage fault]. Piner takes places in AZ, which adopts UCATA in 1984 and then amends it: Abolishes J&S Liability hence contribution is abolished. o Better for s only liable for your percentage you are at fault; no NEED for contribution. Courts have to allocate responsibility to the phantom tortfeasor. o If you dont have this and is 30% at fault and s A & B are each 35% at fault and has $100,000 in damages, the MAX can get
33

from either is $35,000, regardless if one of the s is insolvent or truly phantom tortfeasor- the remaining $35,000 is uncollectable. o Phantom Tortfeasor: Accident in intersection collision; is a passenger in As car, who is speeding. B ran a red light. B is a hit & run driver and never found. can sue only A, B is phantom tortfeasor. Normally, a judge would say that you cant apportion % of fault to phantom, must apportion all liability to A, which goes against the abolishment of J&S liability. Abolishment of J&S liability also abolishes contribution; only liable up to your portion of fault. Purpose is to make sure deep pocket isnt get put on the hook for 100% of the damages. Therefore, tell jury to allocate a % of fault to the phantom. Piner argues he should recover based on holding in Holtz case, which allowed recovery based on : (1) asserted claim against all wrongdoers without having the burden of proving the extent of damage or injury caused by each; shift burden of apportionment to s; and (2) If they cant prove which one caused which portion, then all s are J&S liable treat like regular indivisible case and can hold either one liable. Holtz solved problem by shifting burden to A &B and applying J&S to allow to collect from either , even if s cant sort it out. However, with amendment, they abolished J&S liability. o However, burden is still shifted on s to determine who caused which portion. Court says comparative fault compares FAULT, NOT CAUSATION. You already know the s were each a CAUSE, the issue is now who was more at FAULT. You have the and s A & B. Jury says A was a low-speed collision, is he at fault as much as B (high speed collision + drunk)? How are they likely to assign fault divisions? Fault percentage might be 20/80%. If you have abolished J&S, then P can collect 20% from A and 80% from B. How do you justify shifting burden of apportionment and then allocating it out? Only SEVERALLY responsible up to your percentage of FAULT, not causation. If jury is allocating between 2 s who are both 100% the cause, they are allocating who is more at fault. Who behaved MORE unreasonably? has to prove how at fault; has burden of proof on breach. o then bears burden of proof on whether they breached or not. Why does Court allow to recover here? RATIONALE: (1) Legislative intent behind abolishment of J&S liability is to cure deep pockets problem. (2) Legislature did not intend to deny all recovery due to inability to allocate portion between s. (3) Main reason: Text of statute says you are comparing fault, can only be liable up to % of fault; not causation. Never going to be more responsible than their percentage of fault.
34

Apportion of fault = comparative fault Ultimate outcome: s A &B cant tell who caused injures; jury told to apportion damages based on fault. If you had J&S in this scenario, could collect 100% from A. Under this new approach, gets past causation problem, and most A would be liable for would be 20% of s damages. Solves causation problem in theoretically divisible but practically indivisible injuries. Incentive is to get all of the evidence out. Shifts BOP to s to apportion; if cant be apportioned, each is severally liable up to percentage-share based on fault.

PRE-EXISTING CONDITIONS o Follett v. Jones: Get into a collision, and died 17 days later. When he was in the hospital X-rays revealed he was also dying of lung cancer, unbeknownst to him. Jury found negligent in causing the accident and that his negligence was the proximate cause of death. Causation problem is attributing portion to accident in proportion to preexisting conditions. What contribution did accident make in ultimate death of ? o TYPICAL CAUSATION RULE: You are responsible for what you caused and ONLY what you caused. From a damages standpoint: Only responsible for the value your negligence took from the . with terminal lung cancer might have a different value to their remaining life than there is to a value to someone with a remaining life without cancer. o RULE: If pre-existing condition is ALREADY ATTACHED to the , then you take it into account in deciding what it is the caused and what the damages are. If the pre-existing condition is NOT attached, & there was a way of avoiding it, then we dont take it into account when deciding damages and causation. o When has pre-existing condition, how is value taken into account? only liable for what they cause. Follett v. Jones. Value of life that took was less than it might have otherwise been, so only liable for extent they shortened the life of what was shortened anyway. o Lancaster v. Norfolk & Western RR Co.: worked for multiple foremen, and under each he was harassed one supervisor threw a sledgehammer at him, another touched him inappropriately, another threw a pick axe over his head, etc. 4 different bosses. Causation Issue: Statute of Limitations- multiple occasions starting at least in 1975, which takes some of the early incidents outside the SOL; can only sue on the later ones. (Was it caused by earlier harassment or later harassment thats subject to the suit?) Other Issue: What is ultimately responsible for? What did they cause as opposed to whats attributable to a (1) pre-existing condition (vulnerability to schizophrenia), and (2) negligence by other actors outside the SOL? What did this (Tynan) actually cause such that the RR can be held responsible for damages to the ? Expert testimony: Mental illness was going to happen either way (s defense) Judge says this does not get them off the hook. Applies Eggshell Skull Rule: You are responsible for the damage; you take victim as you find him. Responsible if they suffer greater harm than expected. Responsible if you cause more harm than would usually cause. (Applies in intentional torts too; like if you batter a hemophiliac in an elevator and he dies)
35

Rationale: Tort is meant to redress all harms; deserves compensation. In order for compensation to be effectuated, you have to give the the value of what was taken from him. If victim is highly vulnerable, tortfeasors bad luck. No discount to average damages. o If victim is less vulnerable, would be lucky; no negligence without damages. If eggshell rule is knocked out, there would be imbalance & underdeterrance of negligence; sets optimal level of deterrence of negligent conduct. Under rule, its irrelevant would have gotten schizophrenia; last act of harassment was the triggering case. o BENEFIT to ? He is only responsible for the amount to the damage HE caused. o Victim is less valuable because they were going to have illness anyway. o Reduces damages because of pre-existing condition, even though it is the condition that makes him liable in the first place. o Damages are for when the schizophrenia manifested itself due to the until the schizophrenia would have naturally manifested itself. Dillon v. Twin State Gas & Electric Co.: Boys walking along bridge, 1 falls and goes to grab wire and is electrocuted. It is clear wire was hung negligently, and shouldnt have been that close to the bridge railings. What did they take from the boy? 2 seconds of life. He was going to fall anyway. o Value of what they took due to the fact that he had a condition that had already attached and was inescapable was that theyre only liable for value they took, which was only a few seconds at most. o Pre-existing vs. Eggshell = this was a pre-existing condition that would have happened anyway. Usually has burden of allocating causation between pre-existing condition and negligent tortfeasor. How do you satisfy the burden? o Apportion between pre-existing condition & negligent tortfeasor. Blatz v. Allina Health System: Heart failure; ambulance called right away but there was an alleged negligent delay in the ambulance getting to the house. s claim: ambulance should have been there 6 minutes sooner; had they gotten there, they allege they would have been able to save her life or her chances would have been improved. Court brought in some expert witnesses that testified if ambulance had gotten there she would have had a better chance and wouldnt have gotten brain damage. Ambulance testified when they did get to the house, immediately her heart started beating again. Had they not breached their duty of reasonable care she wouldnt have suffered brain damage. Basis of appeal: Pre-existing condition, already suffering from some sort of heart problem. Jury instructed that burden of proof is to the at-fault ; but it should be on the .
36

Error: dont treat it as an indivisible injury case when allocating between PEC & negligence of - the rule is that the has the burden or allocating causation between PEC and negligent tortfeasor (see above). Rationale: Cant appropriate damages to a cause. If you shift burden to youre transgressing against normal burden; normally has burden of proof in showing s negligent. Not fair when one of the causes is totally innocent and is related to the . EFFECT: Although burden of proof is kept on the , he still can recovercan bring evidence of expert testimony, etc. Let jury decide. To solve s causation problems: Courts comfortable solving this only where justice is demanded & dont want to send home empty-handed and good reason to make it fair to bend normal causation rules. LIMITED purpose substitutes. Not going to solve causation problem every single time. o

PROXIMATE CAUSE - LEGAL CAUSE Every physical cause has multiple effects (like ripples in a pond; chain reaction effect). Doesnt make sense to hold EVERY minor, trivial cause liable. o At some point liability must be cut off. Underlying goal: Fairness; at some point liability has to stop; unduly burdensome on s; would never be able to escape the consequences of a single negligent act. Policy & proportionality determination. Assuming a has been negligent and that negligent is the cause in fact of harm to a , should the be held legally responsible for the 's conduct as a matter of justice and legal policy? o The issue is sometimes stated in terms of the scope of the 's duty of care: did the 's duty of care extend to the avoidance of the harmful consequences? o Often involve a chain of events, some of which are intervening events, i.e. events which come into active operation in producing the 's harm after the has acted negligently. The question to be resolved in these cases is whether one or more intervening events should be deemed a superseding cause of the 's harm, i.e. a cause which exonerates the of liability towards the .

Two Main Tests: o Directness Test: (links in a chain); asks if negligence of so closely linked w/ damages suffers that its fair to hold the responsible for those. (i.e. proximity, are damages suffers proximate in degree to the negligence, etc.) Criticism: Arbitrary; hard for jury to determine. Not much predictability. o Foreseeability: Take the breach and ask yourself: when a is ABOUT to breach, what are the foreseeable types of harm?; given that breach, what are the foreseeable types of harm from that breach? What are the ordinary things youd expect from the breach? Breach because it creates unreasonable & foreseeable risk. Given the risks/harms we envision when we made that breach, was what happened to one of them? Similar to negligence per se effect.

37

The ultimate question raised in proximate cause & negligence per se is: Was the rule of law violated by the designed to protect people like the against harm of the sort the suffered?

Ask: (1) Given the breach, what were the foreseeable types of harm? (2) Were the damages the type of harm that was foreseeable? Criticism: easy to overstate certainty/predictability. Provides GUIDES for predictability.

Breach vs. Proximate Cause: Foreseeability o Basic test for breach: Did behave as a reasonable person under the circumstances? Foreseeability under RPC: Reasonable person can foresee the risk their actions would cause. Another way of getting to this concept: Learned Hand Formula (B<PL) o PC: Example- Reaching down fumbling with radio, and crash car into building. Inside building is a 3-ton bomb that blows up a city block. Negligent in fumbling with radio; unreasonable. Are you liable for all of the damage that ensued? Obviously breached by looking down at radio. Reasonable person wouldnt fumble with the radio because they could envision hitting a pedestrian or another car; maybe even crashing into a stationary object. PC question: Is running into a 3-ton bomb the kind of thing you envision when you fumble with your radio? Given the breach, was a foreseeable type of harm the type of harm that actually happened? Application of Direct Test o In Re Polemis: Extensive fire damage to a ship that resulted when a heavy plank that s employees dropped into the hold of the s ship caused an unusual spark and an unexpected fire. Breach- dont throw heavy objects because you can hit someone; could also break a hole in the ship & it sinks- both are foreseeable- reasonable people could foresee that. Fire, however, was unexpected- fluke. Flukes usually proximate cause issues! Given that there was a breach, whoever who was responsible for the negligence was responsible for the damages the fire caused. Direct consequences test. Make distinction between direct & indirect. Was the fire a DIRECT consequence of dropping the plank in the hole? Yes- therefore, people who dropped the plank are responsible for the fire. How do you determine if its direct or indirect? o Whether court can find if there was any intervening force or cause, or independent cause. o Links of chain metaphor used/ or falling dominoes metaphor. If something intervened = indirect. If dropping the plank was likely to cause SOME damage, liability is extended to injuries for which the would not have been responsible if they ALONE were to be apprehended. Even in a jurisdiction that applies directness, Foreseeability is RELEVANT, but question is where?

38

Breach issue. Cant say it was a breach unless some harm was foreseeable. Wouldnt have been a breach in Polemis unless it was foreseeable in the first place. In proximate cause, you are responsible for other damage as long as it was a direct consequence. Direct test- imposes broader liability. Foreseeability test- imposes narrower scope of liability.

Foreseeability o Wagon Mound I: negligent when furnace oil (relevant because different oils have differing ignition points- furnace oil requires a higher level of heat to ignite than crude oil) escaped from ship into bay, started fire in oil beneath s wharf, who was welding at the time, destroying the wharf. investigates and decides that theres no danger of oil igniting, and they continue to weld. Fire occurred because sparks from the welding operations set fire to cotton waste or rags floating in the water; this in turn ignited the furnace oil. Trial judge finds, in regard to foreseeability: there was no way could have been reasonably expected to know that the oil was capable of being set afire (NOT FORESEEABLE THAT OIL WOULD IGNITE), but s could still recovery because of Polemis case (DIRECTNESS TEST), concluding that the fire was a direct consequence of the release of the oil. Appellate Court applies foreseeability test; criticizes directness test: Too imprecise Too harsh: For it does not seem consonant with current ideas of justice or morality that an act of negligence, however slight or venial, which results in some trivial foreseeable damage the actor should be liable for all consequences however unforeseeable and however grave, so long as they can be said to be direct. o If some damage of ANY kind is foreseeable- youre liable for everything. Needs to be proportionality. o Overstated directness test, however, because this test & foreseeability test are basically mechanisms for CUTTING OFF liability shouldnt be responsibility for every single consequence of your negligent act. Too illogical. o If nothing was foreseeable at the outset, you dont go down the chain to determine if something was direct or indirect (under directness test). o Criticisms of directness test: imposes undue liability that it is too broad & too harsh, and the basis for this is proportionality. Who can really define direct? o Wagon Mound II: s recover. They were destroyed by fire also. So why did they recover if the fire was unforeseeable and you apply the foreseeability test? ships put on different evidence about the foreseeability of fire. Why did A not want to say fire was foreseeable (in Wagon Mound I)? Because there would have been TWO breaches- they would have been liable for contributory negligence for welding on the dock if fire was foreseeable! If guilty of contributory negligence, they take home nothing. o Therefore, no incentive to put on evidence showing fire was foreseeable. Ships in Wagon Mound II werent welding and have every incentive to show foreseeability, which changes the outcome of the case. No need to avoid foreseeability in this case.
39

s in II- zero risk because they cant be found contributorily negligent.

Foreseeability Test & Eggshell Skull o will be liable if some injury of the general type sustained was a foreseeable consequence of s negligent conduct, although the extent of the injuries may be quite unexpected. does not have to establish the foreseeability of either the extent of his harm or the manner in which the harm occurred. If degree or extent of harm is unforeseeable proximate cause; if harm itself is unforeseeable- no proximate cause. Foreseeability & Mechanism of the Harm o Not necessary that should have had notice of the particular method in which an accident would occur, if the possibility of an accident was clear to the ordinary prudent eye. If the and the general type of harm are foreseeable, recovery is usually permitted, even though the particular way in which the harm came about, the manner of occurrence of the harm- may be quite bizarre. Mere fact that the WAY in which the accident happened could not be anticipated is not enough to exclude liability. If the mechanism of the harm is unforeseeable proximate cause; If the type of harm is unforeseeable- no proximate cause. o Doughty v. Turner: Employee knocked something into a large vat of molten liquid; chemical eruption that burned the employee. Type of harm in this event- more than one way to describe it. If youre the who has suffered damages during the explosion, you say only the DETAILS were unforeseeable (the chemical splash v. regular splash). Argue that only the added detail, or mechanism, of how it occurred, is unforeseeable; but the TYPE of harm was foreseeable. would argue that its a different type of harm than the splash would have expected. Say you expected a splash, not an explosion- different type of harm unforeseeable. Whose argument has better weight? Depends what you convince jury/judge. Court held the worker could not recover because the chemical eruption was not foreseeable. How you describe the type of harm, at what level of generality, makes a big difference in the outcome of the cases of proximate cause. Flaming Rat Case: Two workers cleaning metal parts in a small room w/ a furnace in it. Cleaning it with gasoline. Rat gets into gasoline and then goes to furnace, causing a huge fire. Was that breach? Yes. Reasonable people dont clean with flammable liquids. You could say the type of harm expected is reaction with heat (vaporization). Hypothetical: You have a restaurant, set rat poison on back of stove. Breach because it might get into the food. Because of heat, the rat poison blew up. Does it need to go to the jury to determine that the type of harm was unforeseeable? Probably unforeseeable, so no liability. A defendant who is liable for negligently causing a personal injury will generally be held liable for subsequent injuries done by rescue & medical personnel in responding to and treating the injury.
40

o o

An actor whose tortious conduct is a factual cause of physical harm to another is subject to liability for any enhanced harm suffered due to the efforts of third persons to render aid reasonably required by the other persons injury, so long as the enhanced harm arises from a risk that inheres in the effort to render aid. Rescuers: applies in actions against tortfeasors who put innocent victims at risk as well as against those who negligently or intentionally put themselves at risk. Within broad limits, they are deemed foreseeable. Danger foreseeably invites rescue. Firemans Rule: prevents firefighters, police officers, and similar professional risk takers from invoking the rescue doctrine- injuries in this occupation are foreseeable. Suicide- when the negligently hurts a person who later commits suicide as a result of the injuries, the general rule imposes liability for the death only if it can be said that the injured person took his or her own life while insane or during unconsciousness or delirium. If the injured person was lucid when the suicide occurred, this counts as an abnormal voluntary act which insulates defendant from responsibility.

The Unforeseeable Plaintiff o Palsgraf v. Long Island R. Co.: A man, trying to board s train, is pushed by s employee. The man drops a package, which (unknown to anybody) contains fireworks, which explode when they fall. The shock of the explosion makes some scales at the other end of the platform fall down; hitting . sues RR for negligence. Case not about different types of harm, but about different types of PLAINTIFFS. Two different opinions in case represent two views about how to deal with unforeseeable plaintiff. Cardozo deals with DUTY plaintiff is unforeseeable because of DUTY issue. Andrews: Unforeseeable issue gets handled with proximate cause. Where do cut off the scope of liability? Both duty & PC are filtering mechanisms to cut off liability. Cardozo suggests there may not have even been a breach at all: was not a wrong in its relation to the plaintiff standing far away. Relative to her it was not negligence at all. In order to analyze one element in a case, you have to hold the other elements constant & assume they can be made out. Go STEP BY STEP- like a checklist. By the time you get down to proximate cause, you must assume theres a duty, breach, CIF. Cardozo is shaping the facts. Trying to cast doubts on whether there was a breach at all. His doubt about the breach help shape the result he ultimately reaches. However, its not really a proper role of judge to reshape breach issuerole for fact finder. Can a reasonable jury have found breach? 2 ways to describe: o 1-Dont push people on to moving trains. -However, already made the decision to jump on, youre just helping them and it doesnt get any worse. Depending how you view, it could be negligent or not. o 2-Other potential breach- Does a reasonable train operator hold the train doors open when the train begins moving?
41

When you get to proximate cause, you must hold that element- assume theres a breach. Rule for when you have no duty: if no hazard was apparent to the eye of ordinary vigilance, no duty. o The orbit of the danger is the orbit of the duty. With the given act, holding the train doors open (the breach), what do you expect to happen? Passenger to potentially fall (foreseeable consequence). o Helen Palsgraf, do we expect her to be injured? Cardozo describes foreseeable harm as property harm to that passenger. A wrong in relation to his parcel. o If there was a wrong to him at all, which may very well be doubted, it was a wrong to a property interest only, the safety of his package. o Several passengers in vicinity could also be in foreseeable orbit of danger. Where does Helen Palsgraf fall in this equation? Is she in vicinity? o No- shes standing far away. o You expect the harm to be getting knocked down, not another impact from the explosion. o Risk reasonably to be perceived defines DUTY to be obeyed. RULE: No duty to one outside the zone of danger. Holding: Since s conduct did not involve an unreasonable risk of harm to , and the damage to her was not foreseeable, the fact that the conduct was unjustifiably risky of the man is irrelevant. s conduct was not the "proximate cause" of the harm to . Dissent- Andrews view: - Where there is the unreasonable act, and some right that may be affected there is negligence (BREACH) whether damage does or does not result. This is immaterial. The act ITSELF is wrongful. We define breach by whether you behave reasonable under the circumstances; create an unreasonable risk of harm. If we cut off liability, where do we cut it off? You owe a duty when in fact your affirmative conduct creates an unreasonable risk of harm to others. What we mean by the word proximate is that, because of convenience, of public policy, of a rough sense of justice, the law arbitrarily declines to trace a series of events beyond a certain point. This is not logic. It is practical politics. Most famous line of Andrews. Must cut off liability at some point somewhere. Not saying that liability is endless. You owe a duty whenever your act creates a foreseeable and unreasonable risk of physical harm to another; or a risk of property damage. Andrews sees this case as a proximate cause, not a duty, issue, that should be determined by jury, not judge. Andrews test for foreseeability: Foreseeability may have some role in testing proximate cause. o Andrews basically expands duty. o As he applies it, he says there was a little uncertainty how far Palsgraf was; no remoteness in time, little in space; therefore let jury decide. o The problem of proximate cause is not to be solved by one consideration.
42

Meaning- foreseeability isnt the sole test to use. o Unforeseeability is a limiting principle to liability; but eggshell skull rule is an exception: the mere fact that injuries were unforeseeable does not cut off liability. Inconsistent way of looking at things. Underlying Policy: Compensation for deter people from negligent acts. Causation stretches out in a potentially unlimited fashion. Proximate cause: need to know every detail.

Hypothetical: Chauffeur negligently collides with other car filled with dynamite, although he could not know it. An explosion follows. A, walking on the sidewalk nearby, is killed. B, sitting in the window of a building opposite, is cut by flying glass. C, likewise sitting in a window a block away, is similarly injured. A nursemaid, ten blocks away, startled by the noise, involuntarily drops a baby from her arms to the walk. To whom is a duty owed in this scenario, under the Andrews approach? Who does chauffeur owe a duty to? EVERYONE, public at large, if youre driving negligently. Doesnt mean youre going to be liable. Is harm clearly foreseeable to any of them? Clearly A, the person walking on the sidewalk (definitely proximate causephysical injury to him is to be expected.) B: Let jury decide- in the vicinity, but a little removed. Its arguable. Leave it to jury to decide if arguable. C: No proximate cause as a matter of law. No foreseeable harm. Baby: is the harm to the baby foreseeable? No.

SUPERCEDING CAUSE o An actor is not subject to liability for harm, for which a force of nature or an independent act is also a factual cause of the harm, if the harm is different from the harms whose risks made the actors conduct tortious. o Watson v. Kentucky & Indiana Bridge & R. Co. if you spill a big pile of gasoline, what type of harm do you expect potentially? Fire or explosion. Does it matter that it was set by an independent actor causing a cigar? Yes and no. It could be foreseeable that someone would come along and accidentally set a spark; and not realize the danger. But if its an intentional act, that will cut off liability. o Key in the ignition cases: controversial. Are you liable if you leave your keys in the ignition and someone steals your car and then causes injury? Its foreseeable that this type of harm could occur; but why should you be liable for the intervention of a criminal act? Treated differently whether you owe a duty or not- whether its in a high-crime area vs. a low-crime area. o Edwards v. Honeywell: Bakers hired to put in alarm. Fire started in house; Baker notified alarm company. First fire department dispatcher called wasnt in the right jurisdiction. Finally the 3rd one called was the right one and was near the house. Added on an extra 4 minutes until the firemen could come. Fireman, Edwards, died in the fire and sued alarm company for negligence. Potential cause in fact problem. If they had gotten there sooner, would they have gotten the fire under control and the fireman not died? Were the 3 minutes really detrimental? Hard to say. Decided as a duty case; unforeseeable . (Cardozo approach)
43

To treat this as proximate cause case, you have to hold everything else (duty included) constant. Zone of danger - to whom are you responsible to if you dont dial the firefighters quick enough? Policy determination about the incentives created by imposing liability, the cost/benefit of imposing liability, the fairness of imposing liability. Posner says market will take care of faulty alarm problem by giving them a bad reputation; enough deterrence; dont need tort liability. ALL about policy and setting the right incentives. The death of a fireman in fighting a residential fire is a rare occurrence. The problem of proving causation is formidable, and the plethora of potential s makes it difficult for an alarm company to estimate its likely liability even if it does foresee the kind of accident that occurred here. Posner ends up saying its a no-duty case and the effect of this is to knock out the entire category of cases of this sort where a person is injured by an alarm companys negligence where they dont have a contract with that company. All about policy who has the greatest control? What kind of incentives if we impose a duty in this case? Potential liability is disproportionate.

In sum, if the s lawyer can show the court that the s injury is within the array of risks that leads to the conclusion that the s conduct was negligent, the legal cause issue should go to the jury regardless of how unusual the particular accident and/or particular injury might seem. DUTY Duty & proximate cause deal with similar types of concerns where to cut off liability? o How broad is the scope of potential liability that can flow from any given act of negligence? o Duty & proximate cause inquiry are precisely the same; except judge decides one and jury decides the other. o Saying it is a duty or not a duty should be reserved for cases when it comes down to knowing exactly how something happened- usually proximate cause issue. Duty vs. Proximate Cause o Courts use concept of proximate cause to limit liability to the foreseeable consequences of a negligent act. Liability should be limited to the circle of foreseeability, the consequences that the could reasonably anticipate at the time he acted. Bars liability for unforeseeable harm. Take something away; cut off liability due to foreseeability reasons. o Courts use the duty concept to deny liability for consequences that are foreseeable. Imposes policy limits on liability for harm which is foreseeable. Duty, specifically no duty rules: based on non-foreseeability based policy reasons. Normally you dont have a duty to avoid negligently inflicting emotional distress. Why? Because it is not foreseeable; dont know how sensitive someone might be. BUT CANT PLEASE EVERYONE variability in personality; freedom of action; dont have to tiptoe around actions to cater to most sensitive people. It is foreseeable you will upset someone throughout life; but
44

liberty would be constrained if you had to gear your actions toward all sensitive people. Policy arguments that dont have much to do with foreseeability can determine duty.

General Principles: o Heaven v. Pender Principle: similar to Restatement (Third) of Torts An actor ordinarily has a duty to exercise reasonable care when the actors conduct poses a risk of physical harm. o Applies when actor acts and physical harm is treated differently than emotional harm. o Most of the time duty is an easy question. No-Duty or Limited-Duty Rule: exceptions to the general rule. o Donoghue v. Stevenson: Man finds half a snail in his beer. Did manufacturer of beer have a duty to consumer? You need a relationship to have a duty. o Risk-creating behavior is what creates a relationship. Foreseeability o You owe duty to party that is closely affected by the act and that could foreseeably be injured- plays a role in duty principle and proximate cause issue. o When it foreseeably poses a risk of harm to another owe a duty. o Palsgraf- Cardozo says no duty to unforeseeable plaintiff. Flipside- you owe a duty to FORESEEABLE plaintiff. Do you treat this like proximate cause? Owe a duty if your actions foreseeably create a risk, and then does it extend to proximate cause? Yes: Just because judge said yes, it was foreseeable (duty), doesnt mean jury would also (proximate cause). o Duty- draws a circle- you owe a duty if your act creates a realm of potential people who could be injured. If there is a physical risk of foreseeable harm, you create a relationship/duty to anyone within the zone of physical harm. Proximate Cause- is the plaintiff within the circle, or outside of it? Bottom Line: Anytime a situation comes up where a person is engaged in behavior or acts that creates a risk of physical harm to someone, then there IS a duty in this scenario. (Unless it falls into one of no-duty rules).

If you have a fact pattern w/ an unforeseeable problem, you can analyze this under duty stage or under proximate cause stage; because depending on jurisdiction it could come up with duty or proximate cause. o When analyzing, if you do it under proximate cause, put a * at duty and say you will discuss it at proximate cause; or vice versa. o If you discussed it already at duty, at proximate cause mention that you already discussed it at duty. Privity of Contract o Winterbottom v. Wright- involved a defective stagecoach; wheel broke off and driver was severely injured. Tried to sue the maker of the stagecoach. (3 partiesmaker/repairer, stagecoach company, driver)
45

o o

Driver of the stagecoach barred from suing the maker of the defective stagecoach. The contract only applies to those who enter into the contract. Those in a contractual relationship: you may have duties and obligations under it, but third parties who contract arent owed any duties. Only person who can recover is person who has privity of contract. Duties only run to person with whom you have privity of contract. Once its out of their hands, they lose control of product. Point is to limit liability. Worried about specter of unlimited liability. Bright-line rule. However, another rule could be established that deals with certain circumstances. A limiting principle could be applied: Argument: When you have a product (like a stagecoach), purchaser tends to be a company. User tends to be a consumer (driver). Does it seem fair through privity to cut off liability to the person who is most likely to be in the zone of danger and be injured if the product is defective? Limiting principle: Can cut off liability through principle of foreseeability. Privity K: No duty rule. Get rid of no duty rule and determine duty based on foreseeability rule. Privity is no longer good law. No duty rule has been replaced by foreseeability. Bush v. SECO Electric Co.: No duty rule in this case: Acceptance Rule- Accepted Work Doctrine: Once you accept product, you are not liable to anyone outside the privity of contract. Cuts off liability once they inspect & formally accept contract- not responsible for any defects. provides the wiring on conveyor belts- they were arguably negligent in how they did the construction work on the conveyor belt because they failed to put a yellow emergency stop button on the machinery- takes longer to stop in cases of emergency. Once employer accepts contraption from SECO, then the liability of ends. Only and employer are in privity of contract. Temporary employee, , injured by conveyor belt and seeks to get damages from - manufacturer of defective conveyor belt. predicts that the Supreme Court would overrule the privity-based acceptance rule for person injuries in favor of a negligence standard rooted in foreseeability. However, acceptance rule was upheld. Stopped at middle line: Blake v. Calumet Construction Corp. creates 3 exceptions to acceptance rule: (1) Dangerously defective: if the work is turned over in a condition that has a propensity for causing a physical harm to foreseeable third parties using it in reasonable expectable ways. (2) Inherently dangerous: (doesnt apply here) - applied only to dangerous activities rather than conditions or instrumentalities, not the machine itself. (3) Imminently dangerous: if it is reasonably certain to place a life or limb in peril. (BLAKE) o Sounds like foreseeability. Owe a duty to those who are going to be foreseeably injured by your construction. Reasonably certain. o trying to use this to get around acceptance rule. Exceptions are so broad they arguably swallow up whole rule.
46

Bush remanded in light of Blake decision.

Duty to Act: Misfeasance vs. Nonfeasance o Misfeasance: Active negligent misconduct actionable. When you engage in acts, (GENERAL DUTY RULE) that create a risk of foreseeable harm to another you owe a duty. o Nonfeasance: Passive failure to take steps to protect others from harm- not actionable. Where you have not acted you do not owe a duty; may be immoral but not illegal. Rationale: Common law is individualistic- about protecting the liberty of individuals. Dont want to infringe on liberty. Many exceptions have been carved out. Relationship COULD be created by volunteering to offer a service- could impose a duty. If you shield the person from getting help from someone else you could be held liable. Argument: Moral duties come first before legal duties. o Good Samaritan Statutes: riddled with exceptions. EXCEPTIONS TO NO-DUTY RULE Volunteer, or Undertaking Exception: (Gratuitous Services Exception) o Owe no duty to rescue someone. If you volunteer to rescue, you owe a duty BUT you cant make the situation worse= Worsening limitation. With this, you can be as negligent as you want, and in gross negligence, even more. Unless you put them in a worse situation you wont be liable. (One of the most criticized doctrines in tort law). Its your actions ONCE THEYRE IN PERIL that makes you liable. If you volunteer to rescue another from a position of peril duty of reasonable care limited to the principle that you wont be liable if you dont put them in a worse position than they were in to start with. Sometimes the more people that are around the less they are willing to help because they all assume someone else will help. If you are a and youre trying to argue there is a duty, your first job is to put yourself on the misfeasance side rather than the nonfeasance side. o If under nonfeasance, you have to come up with exceptions to the nonfeasance rule; like volunteer rule/undertaking, special relationships, creation of peril. o Sometimes this can fall into the creation of peril exception: If the s own negligent conduct is responsible for putting the in a helpless situation, the has a duty to come to the s aid. Public policy may dictate that some relationships do require a special duty. o Inaction: No duty to actually act, but once you act, it takes you out of no duty rule and takes you into exceptions. If I act & create a peril to another duty of reasonable care. o Lacey v. United States: Administrator of estate of a pilot who lost his life after plane crashed sued the Government for negligence, contending that the Coast Guard negligently failed to reach him while he was still alive.
47

Rescue service- duty to public at large- not to any particular person, unless there is specific reliance by that person in a specific event that prevents them to seek help from elsewhere. Duty to public at large doesnt create a duty to any individual: If you called police to come to your house and they came a little too late and got killed, no duty owed- no liability. The Good Samaritan rule imposes the duty to act with due care ONCE he has undertaken rescue operations. The rationale of the rule is that other would-be rescuers will cease their efforts to help once they believe effective aid is being rendered. In this case, the rescue attempt hadnt reached the stage where other would-be rescuers were induced to cease their efforts in the belief that the Coast Guard had the situation in hand. The deceased was in no way deprived of other available help; and there is no tort liability on the Coast Guard. Essence of the rule is that one who volunteers to assist another thereby undertakes a duty of reasonable care not to make the situation worse. Ex. Bunch of fisherman near drowning pilot; say the Coast Guard is coming and leave him alone- go back to shore. Then a duty is owed to the pilot by the Coast Guard. Puts him in a worse position than he was in before. Dont have to verbally agree to rescue- can be evident through your actions. Poison Pit Hypothetical: Walking through forest and see a person has fallen into a pit; which is filled with poison gases. You are the only person around and decide to go into pit and save person. Pull them half way out; decide theyre too heavy and decide to leave. What duty do you owe at this point? Is it triggered here? You already took an undertaking to rescue- duty of reasonable care is owed. However, not any worse off than they were before you came along; going to die either way. Law judges position person was in before. Exception to No Duty Rule: A person who voluntarily chooses to aid a helpless person, and then discontinues the giving of aid, will be liable to the helpless person if the discontinuation of aid leaves that person in a worse position than he or she was in when the aid began. MODERN APPROACH TO EXCEPTION: Once you begin an undertaking, you owe a duty of REASONABLE CARE. Approach depends on jurisdiction. Mention BOTH on exam!

Duty Based on Special Relationship to Victim o Courts often impose a duty to aid based on a pre-existing relationship between the and the person who needs assistance, even if is not the source of the injury-producing conduct. Courts will impose a duty to take affirmative steps to minimize or avert the harm. Duty Based on a Special Relationship to the Perpetrator / Third Parties o Duty to control one person to prevent him from injuring others. Reflects courts decision that accepts a duty of care by taking charge of a person who poses a risk of injury. must be in a unique position to prevent the harm. Example: Parent/child, Employer/employee. o Tarasoff v. Regents of the Univ. of California
48

Poddar becomes obsessed with Tatiana; who doesnt want him. He goes to see a therapist and tells him he wants to kill her. Therapist notifies police that they should confine him. They pick him up, talk with him, think he is rational & let him go. Doesnt go back to therapist after this. Then convinces her brother to live with him and ultimately uses this to find Tatiana and kill her. Family learns in criminal trial that everyone knew he was thinking of killing her but them. They went after police & therapist. Police get knocked out of liability picture because they have a specific immunity from this. Public policy- courts worried people with mental illness wont get due process rights before being involuntarily committed. California Supreme Court has a theory of duty & when its imposed: how can a new duty arise? When is it appropriate for a Court to impose new duties? Prosser: Duty is not sacrosanct in itself, but only an expression of the sum total of those considerations of policy which lead the law to say that the particular plaintiff is entitled to protection. This is a no duty case: normally no duty to control acts of third parties. Need to carve out exception; based on relationships and considerations of policy. Changes the nature of the doctor-patient relationship. Therapist may think his role is to protect patient; but now his role is broader; have to think of needs & interests of public at large. If you have a dangerous patient you owe a duty to the public. As a minimum you owe a duty to the identifiable victim of your dangerous patient. Holding: The duty is triggered when the therapist actually does predict violent on the part of their patient. Other part of duty (broader) when a therapist determines, or pursuant to the standards of his profession should determine, that his patient presents a serious danger of violence to another, he incurs an obligation to use reasonable care to protect the intended victim against such danger. Only triggered when there is a SPECIFICALLY IDENTIFIABLE VICTIM (not public at large) o Specific Duty: not a general duty of reasonable care, but a specific duty to warn. As long as you have warned, you have satisfied your duty. Reasonable care requires you to tell the authorities. Case doesnt deal with liability. Tarasoff says therapist OWES a duty but they may not have breached itcontacting the authorities may have been enough. How do you impose a duty when therapists arent statistically proven to predict violence that well? (Only 50%) But is difficulty of predicting violence justification for limiting duty? Goal is to err on the side of caution. Argument: Net effect of Tarasoff is that many people may not seek therapy if they are afraid their confidences will be revealed. Court didnt buy this argument; applied a cost-benefit analysis. Trend in tort law in the time of Tarasoff- to send more things to the jury; which may have been a reason they expanded duty- to allow it to get to a jury. This trend has since reversed itself.

49

Argument that might distinguish duties of lawyers from duties imposed in Tarasoff: nature of special relationship was therapeutic with a professional trained to deal with mental illness and to predict violent. Tarasoff should not apply to lawyers who are not trained to deal with mental illness and to predict violence; not fair to impose same duties as we do upon therapists.

Duty Based on Innocent Creation of the Risk o When , without negligence, creates the risk that causes injury to the . o An actor who has injured another, even without negligence, has an affirmative duty to render assistance to prevent further harm to the injured party. Based on the risk-creation rationale. o Schenk v. Mercury Marine Division: Guy goes with on a boating trip with friend and woman with whom he had a special relationship. She had waders on that didnt fit, so he gave her his pair, which were big on her. He strapped them on her with a belt. Guys got into boat and then she got in; problem with motor and water started coming in front; friend moved and weight shifted, causing it to tip over; they all fell in water. was able to escape because he cut the straps off his wader. The other two drowned because their waders were too heavy and water got in them. was sued. Once they decided it was a rescue case rather than a misfeasance; they had to turn to exceptions. tries to bring in the exception of a duty based on a creation of peril. Creation of peril also applies when peril was non-negligent. (ex. You non-negligently hit a deer on the road and kept going; you were nonnegligent but must now use reasonable car to fix the situation. Even if you non-negligently create peril, you have a duty to warn others of the peril.) If you negligently create the initial injury misfeasance Misfeasance (Negligent risk creation) vs. non-negligent risk creation (nonfeasance) First exception: Person has no duty to render assistance to another if he is not liable for the initial injury in the absence of a special relationship. If you are liable for injury, then its misfeasance. Second exception: Every person who engages in the performance of an undertaking has an obligation to use due care or to act so as to not unreasonably endanger the person or property of another. This case actually falls under the Volunteer Exception; however, they said it doesnt apply because there was no undertaking; he was assisting her in a course of action that she had chosen to engage in. If you think its a duty to rescue, this could fall under several exceptions. Court thought it was an undertaking case- falls under Volunteer Rule: didnt really volunteer to assist her because this was something he wanted to do anyway; he made her safer by putting her in better waders. Didnt make her any worse off. Did he volunteer to render aid or assistance? No. Was there a special relationship? What kind of special relationship would this fall into? Not husband & wife; in which case the outcome might be different; different duties. Other types of relationships that trigger a duty to aid: common carrier/passenger, innkeeper/guest, occupier of land/public invitee, employer/employee, husband/wife.
50

Alleged breaches: Allowed her to wear waders that were too large; failure to act when she complained they were cumbersome; failed to warn and instruct her of proper and safe use of waders; and failure to warn and instruct her of the proper use of a life jacket. You dont want to describe a breach as a FAILURE to do something (this automatically puts you in the no-duty rule and you have to then find an exception it falls under.) Instead you should say he CREATED A PERIL by strapping her into waders, not that he created a risk by FAILURE to instruct. Creation of peril: arguably he negligently failed to do anything once she was in the water; but this wasnt argued in the case. Majority stresses that she chose to do all of the acts. He acted, but not in a way that increased her peril; therefore he is not responsible. Treat him as potentially putting her in a BETTER position- but that is more a question of breach than duty. Dissent: If you consider this to trigger undertaking rule, did his assistance enhance the peril she was in? Did his assistance put her in a worse position than had he not helped? If he enhanced peril through his actions, he probably had a duty to warn her of the possible consequences of the heavy waders; maybe should have given her a pocket knife; especially because he is experienced. Reasonable care standard. Possible Argument: His affirmative acts of dressing her in the waders & strapping it on with a belt was misfeasance. Creates danger of drowning and increased risk of physical harm. Wouldnt fall into any exception. If he had a duty he would be held to the standard of a reasonably experienced person under the circumstances.

Sponkowski v. Ingham- four cars driving; following experienced front driver to a party; lost control and ended up in ditch; second driver struck tree and died. First driver responsible because he voluntarily assumed the performance of a duty and he is required to perform it carefully, not omitting to do what an ordinary prudent person would do in accomplishing the task. Example: You motion someone to cross street because they cant see if cars are coming and you can. They go and get hit by a car. Did you owe a duty to them? Yes. You undertook to render the service of traffic direction; so if you breached it you could be liable. Farwell v. Keaton- Two boys went drinking & got in a fight; one got hit unconscious, his friend took him home and left him in the driveway, next morning he was dead. Did the teenage boy owe a duty to his friend? (Normally friends are not sufficient enough special relationship to trigger any duties). Court said he knew or should have known of the peril Farwell was in and could render assistance without endangering himself and had an affirmative duty to come to his aid. If he had just left him where he was and never tried to move him into the car, he would not have had a duty. (This is valid precedent in Schenk case, but court seemed to ignore it). Taking him away from public view and not rendering aid seemingly put him in a worse position. Stretches exceptions about as far as you can go. Combined rationale; not clear which exception this falls into. The misfeasance/nonfeasance line is not clear cut. Creativity involved in getting yourself to fall into one of the exceptions.
51

Dissent in Schenk is at least as plausible as the majority in Schenk. Case could literally go either way. Must know arguments for both sides.

Galanti v. United States: Underhill was a witness in a government case against Thevis, a fugitive, and was being protected by FBI Agent King, who knew Underhill was in extreme danger at all times. He warned Underhill not to inspect the property, but he ignored King. King, however, failed to notify Galanti of the potential danger and did not arrange for surveillance of the property. Both were shot by Thevis, and appellant sued the Government for negligence, claiming Kings failure to warn or protect Galanti against a specific, foreseeable danger was a negligent act and the proximate cause of her husbands death. The general rule of nonfeasance is that no one has a duty to warn or protect another person from a foreseeable risk of harm simply because of ones knowledge of the danger. The mere foreseeability of injury to another person does not of itself create a duty to act. claimed all 3 exceptions to this rule were present in this case; but the court found they were not. (1) Prior Conduct Exception: A duty to warn or protect a third person from danger will arise if the affirmatively contributes to the creation of the danger. (2) Relationship with Perpetrator: The legal duty arises only if the failed to exercise his ability to control the foreseeably dangerous instrument (/potential criminal). o Case intersects with Tarasoff. (3) Volunteer Exception: Law enforcement officials may have a legal duty to warn or protect against danger if they voluntarily assumed or incurred that duty to a specific individual.

Summary o (1) No general duty to act for the benefit of another; o (2) Those who choose to act owe a duty of due care in such conduct. Heaven v. Pender Principle Exceptions: Duties to the unborn and duties to avoid inflicting emotional distress. Circle of foreseeability drawn more narrowly. DUTY - NEGLIGENT INFLICTION OF EMOTIONAL DISTRESS Mental & Emotional Distress o General Rule: No duty not to negligently inflict emotional distress. Dont have a duty to avoid infliction of emotional distress. (Dont have to be nice!) o Exceptions: Start with impact rule. Many jurisdictions move away from this to zone of danger rule. o There are 2 categories of negligent ED cases: 1.) Direct infliction of ED; 2.) Indirect ED cases; or bystander cases. IMPACT RULE (DIRECT) o Recovery for emotional distress proper if also suffered physical injury.
52

o o

o o

ED damages often described as parasitic. Even if ED led to a physical illness, recovery was barred. No recovery of damages for injuries resulting from fright or nervous shock or mental or emotional disturbances or distress, unless they are accompanied by physical injury or physical impact. Injury has to be a DIRECT RESULT Doesnt have to be an injury - just an impact! Courts have different ways of defining an impact! RULE Mental distress first & then physical impact- no recovery. Physical impact and then mental distress- recovery. Policies for Impact Rule: Protects from eggshell skull who has a weak mental state. Cant go throughout life being unduly sensitive. Court says policy behind rule: Dont want to open Pandoras Box for fraudulent claims. Dont want people to bring false claims; dont want to open up the flood gates of unlimited litigation. Have to cut off litigation somewhere. Another policy: Medical science cant verify causation. Bright Line Rule: easy to apply to cases of this type; cut off litigation. Courts arent competent institutions to sort out these claims. Protect against people who are extra sensitive. (What about Lancaster? He was especially sensitive- essentially there was physical harm. Treated as full-fledged infliction of emotional harm.) Policy arguments that run contrary to above arguments: Bright-line rules are arbitrary & unfair at margins. Physical impact may not always be directly related to the mental harm. Advances in medical science; changing conditions (technology, social norms, science) require new law. Old ways are unfair and outdated. Courts are competent and can draw a line; can sort out genuine claims from frivolous claims. Arbitrary to deny recovery to people like Bosley. Bosley v. Andrews s cattle strayed onto s farm; ran toward and had a heart attack. o Bull never touched critical in case. Wasnt just damage to , but damage to s property- she got recovery for that. $179.99 for property but no recovery for heart attack. If the bull had gorged , would she have gotten recovery for physical injuries? Absolutely. o Must be a direct result. As part of that recovery, if she had to have her arm amputated hypothetically, she would get medical expenses, pain and suffering (includes element of emotional distress), lost wages, loss of earning capacity, and maybe loss of consortium (relationship with spouse; sometimes with children) Must look at SEQUENCE OF EVENTS. o If bull gorged her and then she had a heart attack, can she recover? Yes. Purpose of impact rule: guarantee that causation works right- actually caused by the negligent event rather than some other cause. Some guarantee of genuineness.

53

The rule permitting recovery for emotional suffering only if it was accompanied by physical injury or physical impact does not insist that the physical injury or impact CAUSE the emotional harm. o At the same time, physical injuries CAUSED by the emotional distress do not qualify. More Examples of No Recovery: Woman finds a centipede in her soup; becomes sick for several weeks- denied recovery. Case might have come out differently if she had actually bitten into the centipede. Explosion occurred which was caused by s blasting; dirt blown on her and she fell to ground; nervous shock. Indirect impact- no recovery. Woman suffers miscarriage from bumping electric car- no recovery; indirect impact.

Zone of Danger Rule (Direct) o Neiderman v. Brodsky- Court thinks impact rule is arbitrary and unfair- extends rule. o Facts: watches son get mowed down by negligent driver; close to son when son gets hit. He gets recovery. Why? He was in the zone of danger. o Court expanded impact rule, citing that the three arguments which would have defeated the appellant are now invalid. The arguments are: 1.) Medical sciences difficulty in proving causation between the claimed damages & alleged fright; 2.) Involves the fear of fraudulent or exaggerated claims; and 3.) The concern that such a rule will precipitate a veritable flood of litigation. The court found each of these arguments invalid and hence broadened the impact rule, holding that if the establishes that the negligent force was aimed at him and put him in personal danger of physical impact, and that he actually did fear the force, the case can proceed to trial. (ZONE OF DANGER TEST) o Rule: Have to have physical manifestation (to distinguish from physical impact) or symptoms of mental distress in order to help guarantee genuineness of the claim. o Zone of Danger Rule: Replaces impact rule. Not always a rule to deal with bystander injuries. o Would Bosley have been able to recover? YES!!! Within the zone of danger with regard to the bull and therefore under this rule she would have recovered. o Premise- if youre close enough to the negligent injury, you are recovering for the fear to yourself (when applied to a bystander case). o Mental distress probably stemmed from fear for his son; probably only in part of fear for himself. Nonetheless they are developing it as a general rule. o What if he is in the window of a building near the street; maybe 40 feet away? Chances are he is not in the zone of danger and cannot recover. This can also be arbitrary- where is line drawn? Is it arbitrary to let you recover if your son is mowed down 10 feet away vs. 50 feet away and you witness it? o Potential arbitrariness and fiction of zone of danger rule that encourages courts to go further to describe this as a bystander case. o Foreseeability with factors- specific foreseeability- constrained by factors.

54

Fiction of test: have to prove you were within the zone of danger; and have to show physical manifestation or symptoms of that mental distress as an added guarantee to the genuineness of your claim.

BYSTANDER/INDIRECT o Zone of Danger- are bystander cases somewhat different? If so, is ZOD type of approach to deal with this? o Sinn v. Burd Mother witnesses death of child right in front of her. All policy arguments come in here for bases of criticizing zone of danger rolecourt says it is arbitrary where it cuts off liability. Court broadens zone of danger rule to Dillon v. Legg Rule: Dillon: sounds like going to general foreseeability test. We are confident that the application of traditional tort concept of foreseeability will reasonably circumscribe the tortfeasors liability in such cases. Seems like no duty rule totally wiped out. But, Court goes into Dillon v. Legg: paradigm case. Adds factors: mother was not in the zone of danger; witnessed accident from front door. Can she recover? Yes. Adds factors to consider in the foreseeability test- specific foreseeability. 1.) Spatial Proximity: Whether was located near the scene of the accident (zone of danger; how close you are) o Where is line drawn? Arbitrary. o If 30 seconds later you drive up and see daughter on ground- is that foreseeable? o There is always foreseeability of grief. o About foreseeability, but trying to limit the circle to people who are most deserving of recovery or more likely to incur harm. i.e. everyone cant recover for watching the Challenger exploding. 2.) Temporal Proximity: Whether the shock resulted from a direct emotional impact upon ; whether saw the accident rather than hearing about it later; 3.) Relational Proximity: Whether or not there was a close relationship between the and the victim. Test has problems though: how do you treat the emotional distress of an innocent bystander? Uninjured bystander problem often referred to as the Dillon v. Legg problem. What if you watch your child get shot on live TV? Make argument that case is similar to paradigm case, emphasize factors of test you are strong on (temporal/relational), and bring up policy arguments (courts are competent to determine which cases are legitimate, flood gates of litigation wont open, etc.) o Other argument: courts need to tightly constrain liability; flood gates of litigation will open; need all 3 factors for liability. Are these factors guidelines or requirements? Just a tool for determining foreseeability; not determinative or definitivecan miss a factor and still have foreseeability.
55

Closer case is to paradigm case (mother watching child get run over) more likely that emotional distress is foreseeable. If it is foreseeable: Owe a duty; it not, no duty. Reasonable person assumes a relative might be around to witness accident. Tort isnt just for grief for death of another; it also compensates for the independent shock of being a witness to a horrible event; something above horror and grief of having a family member killed, by actually witnessing it. Court also adopts another requirement from Hawaii Case: Leong v. Takasaki Leong court attempted to achieve an objective standard; has to be a situation that would produce emotional stress and requires it to be of a nature that would be likely to produce a response in a person of average sensitivities. Takes care of eggshell plaintiffs- people who are mentally fragile and super sensitive. Independent requirement in doctrine that says you must show it must be of a person of average emotional sensibilities. Event must objectively cause emotional distress to person of average sensibility to get recovery. Add on requirement to make it even harder to recover. Have to meet Dillon v. Legg AND fulfill objective rule. Another requirement needed (to limit liability): the development of physical manifestations: Physical manifestation requirement = Strict interpretation or loose Must be a physical manifestation of your emotional distress in order for you to recover; most jurisdictions have this. o Another limiting doctrine.

Armstrong v. Paoli Memorial Hospital Wife gets phone call from hospital that husband was in an accident and was severely injured. Went to hospital and spent over an hour looking at x-rays, but never got to see patient. o Finally her sister saw license and realized it wasnt really her husband; she loses it, goes into nervous shock, etc. Wants to recover. o Sues hospital- they should have checked before they called her. Doesnt get to recover. 3 ways they judge if she can recover: o 1-court says a reasonable person would be relieved if they found out their spouse was not really the one in the accident; a reasonable person of average sensitivity wouldnt respond as she did. o 2-Looked at physical manifestation Had nightmares, depression, insomnia, unreasonable fears about the safety of her family, etc. Court says this is enough (depends on jurisdiction some require something concrete like something you need medicine forCOURTS CAN TIGHTEN OR LOOSEN THIS REQUIREMENT AS THEY SEE FIT TO LIMIT LIABILITY) o 3-Foreseeability analysis based on Sinn v. Burd. Was she a foreseeable ? Doesnt recover because she doesnt satisfy 2 prongs of the Dillon test.
56

She failed to meet the bystander test because she was not related to the actual accident victim and did not have a contemporaneous perception of the accident. Also, the information that the victim was not her husband would foreseeably cause RELIEF, not distress. In some jurisdictions (PA), impact survives as an alternative theory available to a P who fails to make out the elements of the bystander recovery or zone of danger. Negligent Infliction- independent tort? Yes. o Now can stand alone without physical injury o No longer a principle that limits duty; general foreseeability approach to handling problem. o Not an exception or limited way of giving recovery; its its own tort. o Floodgates opened and liability expanded after Sinn v. Burd decision. Opens doors to cases like Armstrong; people trying to recover after suffering grief from things that happen in daily life. o Once courts open door, there is progression toward broader rules to allow recovery for emotional distress. o

DUTY- OWNERS & OCCUPIERS OF LAND If you have something on your property that falls on someone on your property- what are your duties? o Owners & occupiers of land owe limited duties; not subject to the normal operation of the requirements of reasonable care. The occupier of land owed limited duties to guard against injuries to persons outside the premises, and his responsibility to those injured on the premises differed according to the circumstances of the victims presence there. Key concept in determining whether was occupier = possession. o A possessor of land is (a) a person who is in occupation of the land with intent to control it or (b) a person who has been in occupation of land with intent to control it, if no other person has subsequently occupied it with intent to control it, or (c) a person who is entitled to immediate occupation of the land, if no other person is in possession under clauses (a) and (b). Occupiers Liability for Injuries off the Land: Sprecher v. Adamson Companies o Between natural conditions and artificial conditions Not liable for natural conditions- no control o Starting point: no duty Changing conditions change of law Should they alter the rule in light of policy and changing conditions? o Urban vs. rural- more ability to control risk in urban setting Duty of inspection: less burdensome, more fair because of duty to public Other states have been restricted to tree cases in urban settings (13 other states) o Evolution to all natural conditions like landslide Rural trees: not a lesser duty, but lesser standard. Only trees that you knew or had a reason to know were dangerous. Less likely to have a reason to know; reasonable care wouldnt dictate that you inspect them all. o Right of control and ability to control possession of land creates this.
57

Modern trend is to impose more duties that come along w/ right of control = changing conditions. Rationale: It is clear that the traditional characterization of a s failure to take affirmative steps to prevent a natural condition from causing harm as nonactionable nonfeasance conflicts sharply with modern perceptions of the obligations which flow from the possession of land. Possession brings with it the right of supervision and control. Mere possession with its attendant right to control conditions on the premises is a sufficient basis for the imposition of an affirmative duty to act. Therefore the rule of nonliability for natural conditions has lost whatever validity it may once have had. The distinction between artificial & natural conditions should be rejected. Courts are moving toward leaving the common law rule in its entirety and replacing it with a single duty of reasonable care in the maintenance of property, The duty of reasonable care for the protection of those outside the premises against natural conditions applies even in rural areas. Occupiers Liability for Injuries on the Premises DEFINITION

INVITEES

Come onto property that has been held open with an implied representation of safety; 1-Business invitee (everyone recognizes this); going to Publixbusiness invitee. Can impose a duty on Publix because they are receiving a commercial gain; quid pro quo relationship; there to confer a benefit on the property owner. Mutual benefit; and economic benefit on the property owners part. Dual rationale. 2- Jurisdictions are split on whether to recognize category of public invitee. (Property held open to public or some segment thereof with implied representation of safety); Modern development.

DUTY OWED Duty of reasonable care; which includes not only what you know but what you SHOULD have known. -Includes a duty to inspect.

LICENSEES

Someone enters property with consent or privilege; like a social guest. Ex. Firefighters.

TRESPASSERS

One who enters or stays without any right or privilege to do so. Ex. Burglar 58

Duty to warn of traps, or latent hazards; as opposed to patent hazards. -Take the property as the family takes it; no duty to inspect for hidden dangers; no duty to correct any kind of dangers. -Only duty is to warn of traps. -With regard to open and obvious things, right beyond the door if there is a giant gaping hole, no duty. No duty except to refrain from injuring them intentionally, willfully, recklessly, or wantonly

through your conduct.


First exception: Injured children Second exception: When the occupier knows that significant numbers of trespassers habitually frequent a particular and limited part of the land, or when a trespassers presence on the land has actually been discovered by the occupier, most courts hold the occupier to a duty to reasonable care in conducting activities on the premises.

Invitees rights superior to licensees rights because: o (1) As to injuries from activities on the land, the occupier owes the invitee a full-blown duty of reasonable care, whereas the duty owed to the licensee is somewhat circumscribed by the licensees obligation to look out for himself. o (2) As to injuries from conditions on the land, the invitee is owed a duty respecting conditions about which the occupier should have known whereas the licensee is owed a duty with respect to conditions about which the occupier had reason to know. The occupier cant escape liability to the licensee by claiming no knowledge of the risk if he or she knew the facts that made the condition dangerous, the occupier has no duty to discover such facts even if reasonable care would require it. Firemans Rule: Firefighters and police entering premises to assist the occupier get classified as licensees and find their protection against injuries limited accordingly, on the theory that they are present by virtue of a privilege rather than an invitation Widely criticized. Hypotheticals o 1- A person knocking at the door of your home; on your doorstep; seeking refuge from storm. If you invite them into your house, they are a licensee. Implied consent unless you have no trespasser sign. o 2- Yarn man mowing your lawn. Invitee. Paying him to do a service for you; mutual benefit. o 3- Aunt coming to visit you for Thanksgiving. Licensee (relative coming over) o 4- Your teenage son and his girlfriend: Son= Licensee He invites girlfriend in while youre not there= Licensee o 5- Person at convenience store not buying anything but using public phone. Invitee; still an economic benefit but not profiting from phone. o 6- Person window shopping at your store. Modern trend Invitee; dont have to necessarily buy something to be a customer. May not be conferring an economic benefit at the moment; but you can get one from people potentially shopping; implied representation of safety; could lead to mutual benefit. Other argument- still a licensee before they had intent to confer an economic benefit. o 7- Shoplifter- comes to store; hides merchandise in coat; slips and falls.
59

Trespasser or invitee; could bring someone in with consent and then revoke it based on what theyre doing.

Dont have to have same status on every area of property. o Example: Publix- shopping; go into door that says Employees Only to use bathroom; go into back. Trespasser; dont have consent; other argument: customary for people to often accidentally go into back; if there is implied consent, arguably, maybe they are a licensee and not a trespasser. If they agree to let you go into employee only area, then you are a licensee. When should landlord be liable for criminal activity on premises? o If landlord has actual knowledge of large history of criminal activity on the premises. Demonstrated pattern of criminal activity. o How is this standard created? Judicially created unless overridden by Legislature. o General Rule: Willing to impose a duty when you have a specific showing of foreseeability; showing a pattern of activity can be very important to form a basis for imposing a duty. Specific foreseeability is important to get yourself out of the general rule that you dont owe a duty to a third party. If there is a specific pattern then you do owe a duty. Stitt v. Holland Abundant Life Fellowship o Facts: tripped over concrete tire stop in church parking lot; sued church for negligence. Is there a duty here? o First question: STATUS QUESTION: Was she a licensee or invitee? Not a trespasser; there with consent. What was she doing on property? There for bible study. Does it matter if she was a member of the church? No. Duties to her are same as to members of the church. Church members licensees- in same situation as a social guest in a home. o Rule: Take premises as they find them; therefore if something is open and obvious, then there is no duty to fix that; only a duty to warn them of HIDDEN DANGERS they may not be aware of. o However, she argues she was an invitee- 2 kinds: business and public invitee. She argued she was a public invitee; because business invitee is normally there to convey some kind of benefit on the property (like shopping at Publix). Public invitee- member of public invited to enter land for a particular purpose. Argument that she is an invitee loses. Some jurisdictions wouldve allowed this. Could you make an argument she was a business invitee? Argument: Choose to contribute voluntarily to Church as a business invitee. People could still convey a monetary benefit on Church. Just because Church finances itself with donations doesnt mean its not a business. Is religion a material benefit? Could be providing a service. o Here, woman is public invitee, but is denied recovery. Why? Dont want to impose a liability on Church; a nonprofit business.
60

Court denies Restatement rule that many other states follow. They reserve invitee category solely for commercial business-type activities. Why? Only fair to impose a duty when you are getting a monetary benefit. Part of movement to contract tort liability instead of expand it. If a synagogue is financed by dues that are assigned to you- is argument stronger or weaker? Are you an invitee because you pay dues? (Fixed payment) Yes- you paid and now you are receiving benefit you paid for; mutual benefit. Why does court want to attack duty instead of breach? No duty as a matter of law or no breach as a matter of law: Trying to impose a duty for everyone in Michigan; broad category; generalized for every case of this type. This is a move to make sure to set a firm boundary for premises liability. Want to make a statement that goes broader than this case. Breach determination doesnt set any kind of precedent; its situational, based on individual factors of case; hard to generalize determination. Dissent: Thinks Restatement 332 should be adopted: (1) An invitee is either a public invitee or a business invitee. (2) A public invitee is a person who is invited to enter or remain on land as a member of the public for a purpose for which the land is held open to the public. Creates an invitee status that does not depend on a commercial purpose. 3) A business visitor is a person who is invited to enter or remain on land for a purpose directly or indirectly connected with business dealings with the possessor of the land. Some states absolve the landowner from liability if the condition causing s injury was open and obvious.

Hypotheticals o 1- People who host Tupperware parties; invitees or licensees? Business invitees Rationale: 1. Because there is a mutual benefit there is a reason to impose liability; get something give something. With a benefit comes a responsibility. 2. If you are preparing your property for people to come on it and do business, there is an implied representation that you made the property safe for patrons. Traditional underpinning rationale of business invitee rule. o 2- Go shopping, child in shopping cart. Status of child- your status extends to them- at LEAST licensee. o 3- Come to Publix; stuff meat into jacket to steal it. Trespasser no duty. What if they buy one thing? = Still a trespasser. o 4- Thief that falls through a roof and gets hurt? Happened in California, they got rid of statuses and just imposed general foreseeability to determine duty; how foreseeable is this happening? Usually owe no duty to trespassers; but creates an argument that you COULD. o 5- Public museums where you COULD donate? Could be a public invitee depending on jurisdiction; existence of voluntary donation probably doesnt change status; still no purpose of monetary benefit.
61

DUTY TO PROTECT AGAINST THIRD-PARTY TORTS & CRIMES Criteria for legitimacy of a newly created no-duty rule: o The judge should acknowledge that a new rule is being created; o The new rule should be justified on the basis of principle or policy; o Something about the lawsuit must be unusual enough to call for special treatment; and o The rule must be clear and broad enough to cover an identifiable class of cases. Courts have created new no-duty rules in connection with liability of s who are alleged to have done something or failed to do something that caused the to be harmed by the crime or intentional tort of a third person. Stagl v. Delta Airlines, Inc. evolution of duty principles o Should courts say there is no duty and not let jury get hold of it? Integrity of no duty rule (no duty to protect against third party). o Facts: 77 year old woman hit by a bag while waiting for luggage after a flight; broke hip. Issue: Is there no duty to protect from actions of third parties, or is this a case where affirmative acts of airline created an unreasonable risk to her? No duty rule, Heaven v. Pender principle? o Essence of breach here was not failing to give instructions to everyone to stay away from elderly people, but the way they designed the pick-up area- could have zoned, roped off areas. She argued that how they set up area created a duty. Alternatively, also argued they have a duty to protect her from foreseeable actions of third parties; Not far-fetched; this kind of behavior happens all the time with impatient and rushing airline passengers- specific foreseeability of this very thing happening justifies imposing a duty. o Comparative negligence argument? Could have waited until everyone cleared out to get her luggage. o Judge uses this case to discuss what the appropriate role of judges is as to assigning duty & proximate cause. Says this is a case that falls under Heaven v. Pender. Leaves it to a jury determination. Says there is risk-creating by third parties; so left to jury to decide proximate cause. o Many courts obscure the separate issues of negligence cases (duty, proximate cause and standard of care) to combine and state them together in terms of whether there is a duty to refrain from particular conduct. McCarthy v. Olin Corporation - 2nd Circuit- Federal o If they dont know how state would rule, what could they do? They can ask the NY highest court whether they would recognize a duty in this case; rare for this to happen. However they dont decide to do it this way; they just determine that under NY Law it is clear they wouldnt find a duty in this case to protect another from the criminal activities of third parties. o Facts: Ferguson boarded a train and shot people; used bullets called Black Talons, manufactured by Olin; brought suit because they said Olin was liable for damages because bullets were of a special kind that caused additional damage.
62

o o

o o o

Said they had a duty to protect from the criminal behavior of third parties by not marketing these bullets or selling them to the public. Why dont they just go after the criminals? = going after deep pocket. Who might be interested in financing litigation of this? Gun control organizations; maybe can use tort liability to cut off weapons of this sort; if cant be done through legislation. Why no duty? Foreseeability: does the court here believe that criminal misuse of the bullet is foreseeable? Yes. But if we cut off duty, it must be for some other reason. If we say there is a duty here, it could result in unlimited liability. Criminal misuse of ANY bullets is foreseeable. Extension of rule beyond this case If you were an ammunitions manufacturer, you would probably go bankrupt. Just saying there is a duty doesnt mean youre liable, but it lets it go to trial. Normally, duties to protect against the criminal behavior of third parties are based on special relationships- normally there is some ability to control the behavior of the third behavior. Here, manufacturer had no ability to control Ferguson, so it would be unfair to impose a duty. Argument: Can impose a duty, but it can be constrained- can only apply to marketing the bullets to the general public beyond law enforcement- doesnt send wrong signal, can still have bullets, but because they have such wounding potential, it is only those kind of ammunitions that the duty is meant to apply to. Courts are competent institutions and can constrain the duty rule. Liability comes with marketing the bullets to the general public. Can try to limit duties (like in Tarasoff- expanded duty but put limits on it- only a duty when patient poses threat to specifically identifiable victim). Another reason not to impose a duty then goes to breach; which is a jury question. Jury may not find them liable. However, they might feel sympathetic to the and find liable- therefore courts are hesitant to impose a duty. Dissent: Says it should go to NY Court of Appeals and that they shouldnt be making the decision. Duty needs to be reserved for the kinds of cases where you can announce a rule with enough breadth to cover a category of cases. Precedent cases: Landlord/tenant duties owe a duty when there is foreseeable criminal activity; a pattern of criminal activity- clearly held in NY. Also when there is a public policy to impose a duty to protect against third parties. Tavern owners have a duty to his patrons to protect them from personal attack when he has reasonable cause to anticipate conduct on the part of third persons which is likely to endanger their safety. Bar fights are common; therefore they have a duty to protect patrons from harm that can result from them. Exceptions to no duty to control rules: (Dram shop) liability- tavern owners may not sell alcoholic beverages to a visibly intoxicated person. Ability to control is different: if they dont provide person with alcohol they cant cause as much harm- arguable special relationship/ability to control; whereas in this case there isnt. Argument: Later fact that they stopped selling bullets to public shows that they might have foreseen a danger and CAN control behavior.
63

What

No hunting purpose for these; advertising specifically targeted for sadistic-type purposes. But some people ARE using it for simply targeting purposes; also, its legal- 2nd Amendment right to bear arms. Just because it hasnt been criminalized, doesnt mean it shouldnt be tortious. If you said the sale of the bullets were CRIMINAL, what would be the effect? The retailer would stop selling it completely. If you said it was tortious; say there is a duty to not market your product in such a way that can be particularly harmful- COULD give effect of them no longer selling bullets; but could mean it forces them to internalize cost of product; maybe put in tighter controls. about cigarettes? Lead paint in toys? Who is liable? Might be fair to impose liability if you fail to warn people that your product causes harm and you are aware. Cigarettes = everyone knows it causes cancer; so how can you impose a duty? Negligent marketing claim.

KFC v. Superior Court: Imposing a duty to comply with a robbers unlawful demand to turn over money in order to protect others would be against the states policy of allowing owners to defend their property. o By framing the issue as a question of duty, the majority usurps the jurys historic function in a negligence case to determine the reasonableness of s conduct under the surrounding circumstances. o The focus of the duty analysis should not be on the details of a s conduct; but whether a must take steps to avoid causing a given type of harm to the victim. What steps a must take to avoid the harm is a question of whether the has met the standard of care by acting reasonably under the circumstances; also, it is a question that arises only after a duty has been found to exist. o Three good reasons why negligence law has allocated the judgment of the reasonableness of a s conduct to the jury as a matter for case-by-case determination, rather than having courts, under the rubric of duty, establish as a matter of law fixed and unvarying rules of conduct for various categories of human activity. 1. Irreducible variety of circumstances which may surround an event that causes harm to someone. 2. Leaving the question to a jury allows successive juries to reassess what precautions are reasonable as social, economic, and technological conditions change over time. 3. Jury brings a wider array of practical experience & knowledge to that task than does a judge. DAMAGES What makes damages in tort law different than damages in contracts? o Contracts damages are more like trying to fix the breach- expectancy puts you in position you expected to be in as contract hadnt been breached; benefit of the bargain; based on agreements; tries to make you adhere to your promises. o Torts damages: not based on relationship of agreements, but on happenstance of negligent acts; transgresses social norms. Goal: compensate victim, put them back in position they were in before negligent acts. o Hawkins v. McGee- hairy hand case. Had 25% use of hand (starting point), promised 100% use, and then got 0%. Contract Damages- 100%. Difference between what you expected and what you ended up with.
64

Tort law you would get only 25% in hand case- puts you in position you were in BEFORE TORT.

Compensatory Damages for Physical Harms to Property o Example: Automobile Accident- car sitting at red light waits for light to change and gets hit. Person who owns car has number of issues: First: Loss of use. no longer has the use of a car, so first we have to determine what the rental value of the car is. Second: Basic measure of damages for property that is not land (chattel) is the DIMINUTION IN VALUE caused by the tort. Insurance policies often measure the loss by the cost of repair, but that is NOT the tort measure- once repaired, the value of the car is not the same as it was before the accident- resale value of car is severely reduced after an accident = Actual Diminution in Value of car is the real legal measure. Third: Consequential or Incidental Damages. Example: property that was damaged was property used in business (like a machine); so now business is shut down. Loss to owner of property is cost of repaired machine AND cost of lost business = consequential or incidental damage. More than just loss or repair to piece of property, but its the RESULT and consequence of that. Example: Trespass- recover for diminution IN value as a result of the trespass + value of rental needed because of trespass. An owner whose property is damaged is generally entitled to recover for a resulting loss of business profits if the amount can be shown with reasonable certainty. Animals, like pets, are chattels. Measure of damages is the FAIR MARKET VALUE of the animal. The sentimental value or uniqueness of the dog is irrelevant in the calculation. Example: Hit a dog with a car and injure it. Owner takes dog to vet and it costs $1,500. Do you now have to pay the vet bill? Lets say the dog is worth $20. Probably not. o Fair market value is the measure of damages; irrelevant if its worth more to the owner. Compensatory Damages in Personal Injury Cases o 1.) Past: Medical Expenses: what you paid for hospital/doctor/nurses, etc. (Special/Pecuniary Losses) If you are you want higher number; number is contingent on if the has insurance and many other factors. Lost Wages: straightforward if you had an adult working in same profession for a long period of time. (Special/Pecuniary Losses) What if you havent worked yet? Like a student? Argue that student could have made a lot more- not an appropriate measure of earning capacity. 2 Categories: ACTUAL LOST WAGES + LOSS OF EARNING CAPACITY (person not earning the amount they are capable of earning) What if temporarily unemployed? = Loss of earning capacity Pain & Suffering (General or Nonpecuniary Losses) o 2.) Future: Medical Expenses: speculative; what will the necessity be in the future? What will the costs be of those medical needs in the future?
65

Lost Wages Assumptions made for lost wages & medical expenses; will he work overtime? Stay in same profession? What are the pay rates going to be? Pain & Suffering Dividing line: from day you injured to day you are in front of jury & they give you a verdict.

Nominal Damages: a trivial sum of money awarded to a litigant who has established a cause of action but has not established that he is entitled to compensatory damages; awarded in cases where plaintiff does not need to prove damages as a cause of action. Punitive Damages: damages, other than compensatory or nominal, awarded against a person to punish him for his outrageous conduct and to deter him and others like him from similar conduct in the future. Present Value of Calculations o Dollar today worth less than dollar tomorrow o Money has time value = Reduction to present value is the inverse of this- if I have to pay you for your FUTURE medical expenses, not correct economically to make pay FUTURE value of present value. o How much do I have to pay TODAY (present value) if I am going to owe you $10,000 10 years from now? Can only SPECULATE as to what the future inflation rate/interest rates going to be. Interest rates also change over time. For jury to decide; they decide rate to discount value. o Say you know operation in 1995 will be $10,000. Giving him this money in 1975 is like giving him a bonus; giving him the time value of the money for 20 years. Compensating him for future expense; must then make reduction for present value. o Past medical & lost wages = NO REDUCTION FOR PRESENT VALUE SINCE THOSE ARE EXPENSES THAT OCCURRED IN THE PAST AND THE AMOUNT IS FIXED. Rodriquez v. McDonnell Douglas Corp. (1978) o Accident took place in 1971. Past: 1971-1975 (first trial) Future- Post-1978. (Appeal trial) o Facts: worked as pipe fitter. Accident happened; can no longer move; work, etc. Issue: did jury act on passion and prejudice when giving this award to s? o Personal Injury cases will bring in an economist to explain concepts to the jury; he will make assumptions about inflation- how much his wages will go up (to compensate future lost wages, assume he will get raises in the future); future costs of medical expenses (speculative- becomes a battle between 2 expert economists). Each economist will suggest a rate to discount ( will suggest lower rate, will suggest higher rate of discount). o Economist in Rodriguez made estimation that a pipe fitter in 2010 would be making over $300,000 a year. If jury believed this, which it did, Rodriguez gets a windfall. o These same economists in this case actually underestimated future interest rate of medical expenses. Doctors wages & medical expenses have skyrocketed. Calculation of damages requires certain economic assumptions.
66

Pain & Suffering o Attorneys fees are usually funded by a contingency fee agreement. o Rodriguezs wife hired a lawyer and signed a contingency fee agreement saying that if they collect anything on accident, attorney gets 1/3, 40%, etc. of what they recovery. If they recovery nothing, attorney gets nothing. Medical expenses & lost wages are paying out to person ACTUAL ECONOMIC LOSS- actually didnt go to work, actually did have to pay for medical expenses, etc. If you cut into this by a 1/3 or 40%, Rodriguez isnt being made whole. o Importance to client is that money to pay lawyer is coming out of pain & suffering; only way to make whole. o NOT reduced from future to present value; not an economic measure. o Court, particularly in McDouglard, explains that its basically a legal fiction. Money doesnt help restore that person to his former self. Money compensates for loss of wages and medical expenses; but it cant compensate for actual harm- like losing the ability to walk. o Pain & suffering are where damages can become large & speculative- is it reasonable? o Procedural Posture on Appeal Jury erred in awarding excessive damages Standard for excessiveness- shocks the conscience at first glance. HIGH STANDARD OF REVIEW. o Options: Can order a new trial on damages only; new trial; or can do a remittiturreduce the damages without having a new trial. Addittur- so low that no reasonable jury could have reached an amount this low and judge will ADD to it; very rarely used. McDougald v. Garber o Facts: suffered permanent brain injury after negligence by doctors when performing a C-section; awarded damages for pain & suffering and loss of pleasures & pursuits of life. s appealed. o Issues: (1) Is some degree of cognitive awareness a prerequisite to recovery for loss of enjoyment of life (do you have to be aware of loss of enjoyment of life to recover for this)? (2) Is pain & suffering a separate element for loss of enjoyment of life? o Holding: Yes on the first issue, no on the second. Trial court erred in jury instructions; new trial on issue of nonpecuniary damages to be awarded. o Rationale: Recovery for noneconomic losses such as pain & suffering and loss of enjoyment of life rests on legal fiction that money damages can compensate for a victims injury. An award for the loss of enjoyment of life cannot provide a victim with any consolation or ease any burden resting on him; he cannot spend it upon necessities or pleasures and cannot experience the pleasure of giving it away. Loss of enjoyment is a PERSONAL type of harm; have to know you are losing it. How can you be suffering if you are not aware of it? Tortfeasor almost better off if they injure someone to the point where they are brain dead than if theyre just injured. Cognitive awareness is a prerequisite to recovery for loss of enjoyment of life. A separate award for loss of enjoyment of life must be on the basis that such an approach will yield a more accurate evaluation of the compensation due to the , which it will not. o Goal of awards: Put you back in position you were in before tort.
67

Awarding for loss of enjoyment of life would not adequately compensate for this. Hedonic Damages: Damages for loss of pleasure; a legal fiction. Counterargument: Since person is brain damaged, person lost ability to enjoy normal facets of life; objectively denied ability to enjoy everyday pleasures of life. Pain and suffering is NOT separate from loss of enjoyment; LOE is just an element of pain and suffering. If you do award loss & suffering for enjoyment of life, should a jury be instructed to give one award for this and one for pain & suffering? Or should they be instructed on each of them separately? Should it matter whether the jury is separately instructed? Why make the award more complicated than it should be? Court says LOEL is a subset of P&S and there should not be double recovery for separate awards. Therefore, new trial granted because jury instructions were found to be erroneous.

Unit of Time Arguments o Sometimes not allowed. o How much would it take to compensate a who is so severely impaired? Example- $10 an hour for life. Windfall for - would get way too much. o Plaintiffs attorneys sometimes show a day in the life video; what they have to endure to do normal daily tasks & how difficult they have become. These are usually allowed. Collateral Source o Payments or benefits from sources other than the tortfeasor; like from workers compensation, health insurance, disability payments, life insurance, uninsured motorist payments, to compensate the for injuries. o Can also include gratuitous services- like if your spouse provides nursing care; payment or benefit that comes to to compensate for injury from tortfeasor. Subrogation o injures in a car accident; has medical expenses that are paid by s health insurance; say it is 100K. o sues as part of the recovery- judgment that covers the 100K in medical expenses. o If the Health insurer has a right of subrogation (which it usually has), then the has to pay back the health insurance company for the damages they had to pay out to the . o Justifiable because the award usually receives covers more than medical expenses, so will usually still get money. If they didnt, then there would be no point in bringing the suit against the . o What if a health company WANTS the to sue the to recover for expenses? can sign over the right to company for them to pursue claim against for which they have a subrogation right. o Right of subrogation comes from contracts; insurance contract usually has a subrogation clause- CONTRACTUAL right. Collateral Source Rule o The tortfeasor does not get the benefit of the payments the receives from collateral sources. Doesnt this allow the to receive double recovery?
68

o o

100K from medical expenses, 100K from - does receive 200K in medical expensesdouble recovery? If there is subrogation- it doesnt lead to double recovery. But not all collateral sources come with a right of subrogation. If spouse gives other spouse nursing care, gives spouse 1 million to cover costs. How is this fair? Punishment effect of tort suit; but really about compensation. Rationale for collateral source rule: courts more concerned with the tortfeasor not benefiting from the payments of the collateral sources than the potential of the getting double recovery. (Subrogation usually fixes this anyway).

Haynes v. Yale-New Haven Hospital o Application of CSR to underinsured motorist coverage: If A is hit by a negligent tortfeasor, lets say they have no money and no insurance. A has $1 million of insurance, has uninsured motorist coverage- As insurance will cover it. o First party insurance- coverage for YOURSELF; property, etc. Just make a claim when you have to specify damages. o Third party insurance-Liability insurance when a person comes with a claim against YOU- protects you when youre a in a tort suit. Provides you with a lawyer. If a judgment is rendered against you, it pays on your behalf. o Uninsured motorist coverage is FIRST-PARTY INSURANCE- replaces what I cannot recover in tort because the has no money. Pays for what they could have gotten in a suit against negligent tortfeasor. If they are underinsured, as the motorist in this case- had only up to $25,000 in insurance, damages were $650,000- in this case had underinsured motorist coverage which covers the balance; compensates what P could have gotten in tort. o goes to hospital, dont notice she has internal bleeding and she dies- wrongful death action- combined tortfeasor situation- negligent driver & malpractice on part of hospital & surgeon- both acts combined produced ultimate death. Goes after driver first- settle claim for 20K. Underinsured motorist coveragepays $630,000 in damages. Goes after hospital & surgeon- assert that she has already been paid; cant get double recovery. Response- yes but that was a payment from a collateral source, and therefore they cant get a windfall by virtue of already getting payment from a collateral source- CAN get recovery. o Rationale: The amount of an underinsured motorist payment is determined within contractual limits by the measure of tort damages. The question is not whether underinsured motorist benefits are a collateral source; but if they should be treated as one. This case is analogous to that of joint tortfeasors, and the general tort rule precluding double recovery from joint tortfeasors should apply. The only thing is deprived of is the opportunity to recover more than she paid for the insurance policy. Wrongful Death & Survival Actions

69

-Not a new cause of action; just a new way to bring suit

Survival Statute

Wrongful Death Statute

Who sues?

For what damages?

Personal RepresentativeStands in shoes of decedent; brings on behalf of estate. Whatever decedent couldve recovered if hed lived (From time of accident to time of death) Whoever was eligible to recover from estate (by will or by intestacy law)

Personal rep. on behalf of surviving family members specified in statute. Harms from family members suffered by virtue of wrongful death (Pecuniary & nonpecuniary) Family members specified in statute, usually spouse, minor kids, sometimes adult kids, parents

-Occurs when brings suit and then dies due to injuries related to accident. Personal representative brings both actions. Survival - time of injury to death. WD tune if death going forward = compensates different things; gets both. Estate (usually people in estate are same as those in statute).

Combined Statutes

Who recovers?

Survival Action o Makes a tort action continue after the death of one of the parties o If was in a car accident and sues , and then dies before suit is brought to trial. Survival action provides that cause of action continues after death of either party. o What if dies because of something totally unrelated to the accident, like a heart attack? had initiated a lawsuit for negligence action during that time; this suit continues even after death of ; but there is a limit. ***Action that proceeds after death of seeks recovery for time period from time of injury to time of death. i.e. Dec 06 (accident) to Dec 07 (time of death) Get everything you would have gotten in a negligence action- lost wages, medical expenses, pain & suffering, etc. This is only if there is no wrongful death action because the death was caused by something unrelated to the . o Whatever damages that were available in action that couldve gotten if he had lived and brought the action is what the personal representative gets on behalf of the estate. Who recovers? The people who were entitled to recover from the estate. o If they die instantly- cant recover for survivor statute- no pain & suffering. Wrongful Deatho Benefits survivors; old cause of action continuing after death of party; same cause of action. o Brand new form of recovery not seen at common law; purely statutory cause of action for damages caused by wrongful death. o Can recover for pecuniary losses (loss of support) and nonpecuniary losses (such as loss of the decedents guidance and companionship). o Exception to the general rule that one has no cause of action for a tort committed on another. Often said that the purpose of wrongful death recovery it to provide for the decedents dependents, and the statutes usually name classes of survivors that the legislature probably assumed would be dependents. o Creates a right of recovery for people who couldnt recover before. Consequences: statute construed strictly. Not concerned about equity.
70

o o

Personal representative of estate brings action; brings action on behalf of surviving family members who are specified in the statute as being able to recover. (In survival, bring action on behalf of ESTATE) People who can recover very limited- spouses can always recover- ALL statutory. Recover for HARMS TO FAMILY MEMBERS. Measured by loss to survivors theory of recovery; also recover loss of services (=consortium, etc.) Loss of survivors theory and loss of services theory- broadened to include loss of society and sometimes even grief. In what way was family harmed? Lost their bread winner- provided support for spouse & children; traditionally the purpose of this was to replace the support when bread winner was killed in wrongful death. If the was not the breadwinner- same people recover, would just get less money. If decedent had no money recoverers get nothing. Traditionally didnt get loss of consortium, grief, emotional damages, etc. Now its possible to get more, but still limited. Other potential problem: who doesnt get to recover? (Aspinall v. McDonnell) If there are no survivors in the category then gets away from wrongful death. If they have no spouse, no minor children they support, no parents they support (if statute provides this) no recovery. Statute- bright line rule; easy to apply. But there are always cases just on the other side of the line. What if person was engaged and was to be married the next day? Is there a danger of double recovery when a family can bring a wrongful death & a survivor action? Example: gets in a car accident, in the hospital for a year, and then dies from injuries in accident. Personal representative brings both actions. Survival covers from injury to death in hospital. o Cause of action survives the death of the plaintiff. Wrongful death is from time of death going forward. Therefore no overlap; compensate for different things. Damages would go to the estate. (USUALLY people under estate are same as specified in statute)

Aspinall v. McDonnell Douglas Corp. o Coinhabitant cant recover because of strict statute; unadopted stepchildren cannot recover. Only people that can recover are ones specifically enumerated in the statute. Indication that decedent wanted to provide for her He left everything to them in his will. Will doesnt override statute! Still cant recover for wrongful death- not married. Statute says heirs can recover only- which dont include people who you are not married to. AFFIRMATIVE DEFENSES: COMPARATIVE NEGLIGENCE
71

Comparative Negligence o Common Law: Affirmative defense of contributory negligence. o Affirmative Defenses: different than prima facie case for negligence Prima Face: must plead & prove by preponderance of the evidence. Defenses: pleads & proves by a preponderance of the evidence. o How does a win a case? By negating one of the elements of the prima facie case (making it impossible for to prove by preponderance of the evidence); OR can win by proving one of the affirmative defenses. o Contributory negligence- failure of the to use reasonable care for his own safety; which barred from ANY recovery in common law. If failed to use reasonable care- got no recovery. o Courts developed doctrines to forgive of his negligence. Ameliorative doctrines- designed to mitigate the doctrine of contributory negligence Ameliorative Doctrines o Last Clear Chance Doctrine Comes from case of fettered (hobbled- tie legs together) donkey. hobbled his donkey and left it in the road-reasonable person doesnt do that because youre asking to get run over- and in fact, came along driving wagon at excessive speed and ran over donkey. was contributorily negligent. Court forgave the s negligence because it said the had the last clear chance to avoid the accident. Last clear chance takes what would be contributory negligence and says it doesnt matter. o ALL OR NOTHING doctrine. o Wilful or Wanton Negligence (Ameliorative Doctrine) If the s negligence is such that you can consider it wilful or wanton, then you can just ignore the s negligence entirely. This ameliorative doctrine is also All or Nothing. Both doctrines are inconsistent with contributory negligence because you are just ignoring the s negligence, but they were imposed because it was unduly harsh to not allow to recover anything. Comparative Negligence o Courts and legislatures developed other ways to deal with when also at fault comparative negligence. Pure: Amount of damages recoverable to are diminished in the proportion which the culpable conduct attributable to the claimant or decedent bears to the culpable conduct which caused the conduct. Modified 51% System: If the negligence of the was not greater than the negligence of the against whom recovery is sought, the damages shall be diminished in proportion to the amount of negligence attributed to the person recovering. Modified 50% System: If the fault chargeable to a party claiming damages is EQUAL to or GREATER in degree than any fault chargeable to the party from whom the claiming party is seeking to recover damages, then the claiming party is not entitled to recover such damages. Sometimes they are called Modified 50 and Modified 49.
72

Hoffman v. Jones Fl Sup Ct judicially adopts comparative negligence- first court to do so. Several rationales for adopting comparative negligence- all come down to contributory negligence being too harsh. Justifications: (1) Court is competent and has the power to do this because contributory negligence itself was a judicial creation. (2) Court looks at the evidence from other states and says that other states are moving to this; so the norms of society as reflected by what other states have done are relevant to their decision. (3) Fair or equitable to do this because contributory negligence is simply too harsh; and to prove this, they point to the ameliorative doctrines like last clear chance; and also say that doctrines that allow juries to ignore or rebel against it arent efficient. Comparative Fault system: have to have a special verdict form for jury- how much do you find s damages to be? What percentage of responsibility for this accident to you assign to each party? Jury then assigns numbers; up to them to figure it out.

Example 1: is 90% at fault; is 10% at fault. s damages are $100,000 o Pure Comparative Negligence: (NY statute) Reduce damages according to percentage is at fault. = gets $10,000. What if jury said s negligence was 99% and deep-pocket allocated 1% negligence to them; and damages are $20 million? gets to recover under pure. o Modified 51%: (Wisconsin) If negligence of was NOT greater than negligence of whom recovery is sought, can recover damages. = recovers nothing. If s negligence is 51% or above, if 2 parties, its still all or nothing; meaning above 51% negligent gets zero. If s negligence is above 51% it works the same as contributory negligence and they get nothing. If its below the 51% line, gets recovery in proportion to fault. Works the same as pure comparative negligence under 51%. Rationale: above 51%- responsible for your own harms; shouldnt get to recover. o Modified 50%: (Arkansas) would get zero. Same rationale as modified 51%; if s damages are above 50% they recover nothing. Example 2: is 51% at fault; is 49% at fault. $100,000 in damages. o Pure Comparative Negligence: would get $49,000. o Modified 51%: would get 0. o Modified 50%:
73

would get 0.

Example 3: is 50% at fault; is 50% at fault. $100,000 in damages. o Pure Comparative Negligence: gets $50,000 o Modified 51%: gets $50,000 o Modified 50%: gets nothing. has to be less than to get recovery. If is greater than or equal to gets no recovery. Example 4: If was 49% at fault and was 51%; under all 3 systems the would get $51,000 in damages. Do you instruct a jury to the effect of splitting it down the middle 50/50 in a mod 50 system? o If you do, they would probably give them an extra percent; instead of splitting it down the middle, so the could recover. o Different jurisdictions do different things- some think its illogical that it would affect their reasoning; fault is what it is and jury shouldnt play games with the percentages to get to a different result. Set Off o Hoffman v. Jones talks about set off and whether its mandatory or not. o Court recognized there would be complications because of adoption of comparative negligence system. Speculation about set off, however. o Set off- A & B go to lunch; A doesnt have money so B pays- $10. So A owes B $10. Next week, go to lunch again and this time B has no money- A pays; $5. So A just gives B $5 and theyre even. Example: & both injured; both at fault. has $100,000 in damages; has $100,000 in damages; both 50% at fault. So they each, under pure system, get $50,000. If there is mandatory set off, how much money changes hands? Zero. They each owe each other same amount; no money passes hands if set off is mandatory. HOWEVER its not really the or paying- its really INSURANCE COMPANIES. If you dont have mandatory set off, then Insurance Co. 2 pays $50,000 and Insurance Co 1 pays $50,000 and total pay-out all together is $100,000. Set off- just seems logical. o Same hypothetical without insurance company: Each owes each $100,000. cant pay; insolvent- if set off is NOT mandatory- gets ZERO- PAYS $50,000- is not protected. Set off protects if is insolvent. If set off is mandatory- no money passes hands in this transaction. o If two parties are liable to each other in the same suit, each party is entitled to a setoff of any recovery owed by the other party, except that, in cases which one or both of the parties has liability insurance, setoff does not reduce the payment of a liability insurer unless an applicable rule or law or statute so provides. o When set off is not mandatory, courts can still apply it in insolvent situations to protect .
74

Not mandatory and cant apply it in insurance situation. is 40% at fault; 1 is 30% at fault; and is 30% at fault. Damages are $100,000. o Result: It depends on which system you use. o Pure: Doesnt matter; recover no matter what. o Mod 51 or 50: Matters what you compare the s negligence to. Do you compare the s negligence to 1 + 2 = 60%? Or do you compare to EACH one separately; in which case wouldnt recover? Wisconsin: you compare separately; meaning would recover nothing. Minority Rule. Majority: you compare s negligence to combined negligence of 1 and 2.

Complications with adoption of comparative negligence: o Must first determine if its set off o Must then decide if doctrine of last clear chance rule survives (forgives or ameliorates s negligence so it wont be outcome-determinative against ); usually off table once comparative negligence is adopted. Wassel v. Adams o Doctrine of Wilful & Wanton- if s negligence is wilful & wanton, you dont compare the comparative negligence of the . o raped at motel and she charged , owners of motel, with failure to warn her of a potential crime. High-crime area. Hotel owners said she should have known better than to open the door in the middle of the night to a stranger- essentially her fault. Jury said she was 97% at fault; s 3% at fault. They awarded her cost of her to have therapy- small number. o appeals and says wilful & wanton negligence doctrine survives and that s negligence was wilful & wanton; and also says weight of evidence doesnt show that her negligence was 97%. Posner: Weight of evidence is enough to show she is 97% of fault. s were not wilful & wanton. o Wilful & wanton- no rational jury could find the s consciously disregarded the high probability of harm (recklessness) o Basis of saying the wilful & wanton doctrine should survive after the adoption of comparative negligence: If she had sued the rapist, would we have a verdict that looked like this? (97% her fault?)- No. Would there have been an allocated portion of comparative negligence to her? No because it was an intentional tort. Why cant you compare intentional to negligent contact? Fundamentally different. o Is wilful & wanton an exaggerated form of negligence (just quantitatively different)? Or is it qualitatively different in the way that intentional torts are qualitatively different? Argument for maintaining doctrine- it is qualitatively different. Argument for getting rid of doctrine- just extra negligence; once you have comparative, there is no need for wilful and wanton doctrine.

75

Posner compared burden of her looking out the door to burden of hotel hiring security guards- alternatives to hotel owners. Guards were expensive, but in relation to the risk? Cost of security guards- how do you afford this? Raise room rates. o Momentary inadvertence or systematic choice to engage in a pattern of conduct which negligence (breach) is worse? Normally we forgive momentary inadvertence more than the choice to consistently engage in the negligent conduct. Also if they warned her, they wouldve warned her not to walk at night, not to open the door. Warning people might also be bad for business. o Standard of review- if a reasonable jury could have apportioned damages this way given evidence they had, you have to leave it this way- appellate court cant overturn unless there is no way a reasonable jury could have found it this way. Not appellate judges role to be a fact finder. o Why didnt they allocate anything to rapist as phantom tortfeasor? Probably had to do with the jurisdiction. To s disadvantage to have it allocated to phantom tortfeasor- it would minimize her damages from the hotel owners. On the other hand, if they allocated to phantom tortfeasor, and you apply J&S liability to the hotel owners, it could have benefited her in this case. Wilful & wanton doctrine tends to go by the wayside with the adoption of comparative negligence its merely an exaggerated form of negligence, so it is not necessary. o The fact that she was from a small town is irrelevant- dont take this into account in objective standard which is about a reasonable person. If it was criminal law it might be relevant. This is like Vaughn v. Menlove- dont take individual characteristics into account. Can we take into account the fact that this was in the middle of the night to determine if she behaved reasonably? Situational circumstance that can and will be taken into account. Its her duty to come up to the standard of a reasonable person. Avoidable Consequences Doctrine o Dare v. Sobule Motorcyclist hit by automobile that made a negligent turn; but he wasnt wearing a helmet and he died. Damages- $4,089.14 for for wrongful death. Issue: Should his failure to wear a helmet be taken into account when determining responsibility or allocating damages? o Policy Arguments (for not taking it into account): No; because shouldnt have to foresee s negligence. Shouldnt have to go through life worrying someone would be negligent. Wasnt a statutory mandate to wear a helmet. If they had to take it into account, there would be a battle of the experts to determine what wouldve resulted if he wore a helmet. Taking the failure to wear a helmet or seatbelt into account would be a windfall to . Most people dont wear a helmet (like T.J. Hooper). o What about failure to wear a seatbelt? Policy arguments for taking this into account: Car Accidents -> 2nd collision injury foreseeable why reasonable people do wear seatbelts; fair to hold it against them if they dont wear them. o
76

Shouldnt have to wait for the legislature to mandate the usage in order to take it into account when deciding how much recovery a gets. Once legislature DOES mandate the use of seat belt or helmet, it should definitely be taken account when determining s negligence. Juries have to deal with experts all the time- juries are competent to sort out the conflicting testimony. Dont want the paying for incremental harm the couldve avoided. Even though most people dont wear a helmet or a seat belt, doesnt make it reasonable. (T.J. Hooper) Seat belts much more commonly worn today + adoption of seat belt laws. If we take failure to wear helmet / seat belt into account?? How do we do this? (1) Can ignore the failure (approach of some jurisdictions) (2) Treat like comparative negligence- i.e. a percentage reduction in recovery (allocate a portion to the failure to wear a helmet in the overall scheme of things as part of your responsibility for damages and then reduce your recovery accordingly.) Definition between comparative negligence & avoidable consequences: Avoidable Consequences: s conduct was not a CAUSE of the accident. The failure to wear a seat belt or helmet doesnt cause the accident, it just causes the damages to be exacerbated; causes an extra portion of damages after the fact. Comparative negligence: failure of to use reasonable care which causes in part the accident. Someone is speeding and you are rear-ended, not wearing a seat belt. Fairly low-speed collision; if you had been wearing a seat belt there would have been no injuries. How should jury allocate percentage fault between you and person who rearended you? Ignoring seatbelt treats you like eggshell skull- get full damages. If treated like comparative negligence and get percentage reduction, how do you know how much to allocate? If experts dont agree, then how do you put a percentage? Problem. If comparing causation, you didnt cause accident at all, only a portion of damages. Comparative fault doesnt compare damages just amount you are at fault. There is another option: Treat like mitigation of damages Example: Injured in a car accident and doctors tell you that you can get physical therapy to get 100% use of legs back; but dont go. Instead only get 50% use of legs back. Is it fair to hold responsible for damages you could have mitigated? would not be responsible for extent you didnt mitigate damages. Policy behind this: have to put person in position they were in before accident. Require s to mitigate damages so they dont take advantage and dont force to bear costs that could have been avoided by actions taken by the . Want to encourage mitigation. Difference between mitigation & avoidable consequences? TIMING: avoidable is pre-accident conduct that causes a certain portion of your damages
77

Mitigation is post-accident conduct that causes a certain portion of your damages. What would we do if we were going to analogize avoidable consequences to mitigation of damages? What do you instruct jury? Totally barred from recovery for the portion in damages that could have been mitigated; or could have been avoided. Total bar is the rule for mitigation in the vast majority of jurisdictions. Some treat avoidable consequences just like mitigation with a total bar, others dont. Rear-ended in stop sign; would have had whiplash but instead get facial lacerations because you didnt wear a seat belt. is responsible for the whiplash only- not responsible at all for the facial lacerations- those could have been avoided if you had been wearing your seat belt. There can be a bad battle of the experts- how do we know what would have happened if you wore the seat belt? Is it better to ask jury to just do a percentage fault recovery? Which is preferable? All about policy- depends on analogy of mitigation or whether you just want to use comparative negligence. Most jurisdictions probably use a middle route.

AFFIRMATIVE DEFENSES: ASSUMPTION OF RISK Assumption of Risk o At common law, assumption of risk by a was a complete bar to recovery. o CHOOSES OR CONSENTS TO KNOWINGLY & VOLUNTARILY ENCOUTER THE RISK CREATED BY THE S NEGLIGENT CONDUCT. If you jay walk- are you assuming the risk? No, because cars are supposed to be looking out for pedestrians. It is a general risk but not a specific risk created by the negligent conduct of the . You could be comparatively negligent, but not assuming of risk. o Adoption of comparative negligence & its relationship to assumption of risk- how do you define the conduct? When a car is speeding toward you and jump out in front of it, is it assumption of risk or comparative negligence? It is both. Make a conscious choice to chance it. Comparative- failure to use reasonable care for own safety. Assumption of risk- knowing & voluntary risk. o If AOR is a complete bar, but comparative REDUCES recovery, which do you use and what do you call it? Many jurisdictions combine the two into just comparative negligence. THREE TYPES OF ASSUMPTION OF RISK: o (1) Express AOR: Sign a waiver to participate in an activity; also called Exculpatory Clauses. Can also be by a conversation. (Verbal contract- I agree to choose to encounter the specific risk.) Mostly covered by contractual principles; used to waive liability. Sometimes invalidated on public policy grounds. Always construed against drafter of exculpatory clause. o (2) Implied Primary AOR: Kind of activity that has an inherent risk in it; in common law it was treated as an affirmative defense.
78

(3) Implied Secondary AOR: , through their conduct, has chosen to encounter a specific risk created by the s negligent conduct. Example: In a grocery store, oil spill in the middle of an aisle that has been there for 4 hours, you SEE the oil on the floor, but decide to tiptoe through aisle anyway, slip & fall and break hip. Conduct manifests your intent to encounter that risk knowingly & voluntarily. Different than implied primary scenario- not an inherent risk of grocery shopping that youre going to encounter a big oil spill on the floor. You see and know of risk created by and still choose to encounter it.

Two types of Implied Secondary AOR: o (1) Reasonable Implied Secondary: Have to take child to emergency room; use friends car who has bad breaks after she warns you; may be reasonable to use friends car to take child to emergency room. Is this comparative negligence? No- can only use comparative negligence when it is UNREASONABLE. A few courts have argued that is completely barred recovery under this because comparative negligence statutes only address the effect of the s FAULT; the rationale for AOR as a defense is knowing consent to take a risk, not fault. Therefore, they argue that passage of a comparative negligence statute should not affect the affirmative defense of reasonable assumption of risk. However, because under most comparative negligence jurisdictions UNREASONABLE AOR is treated as a form of the s negligence, which reduces a s recovery rather than barring it, it is only consistent and just to treat a s REASONABLE choice to encounter a risk negligently created by the as nonnegligent conduct, which should not reduce or bar her recovery from the . o (2) Unreasonable Implied Secondary: Grocery store example; usually describe it as contributory or comparative negligence. Example- go to friend to borrow car; friend says car has bad breaks and not a good idea; you take car anyway to go play poker. This is unreasonable failure to use due car for ones safety. Assumption of risk but its also comparative negligence. If in a comparative negligence jurisdiction, recovery would not be fully barred. If unreasonable assumption of the risk persisted as a separate defense, then WOULD be fully barred. Most jurisdictions conclude that unreasonable AOR should be treated as a form of negligence and reduce recovery by percentage. Bennett v. Hidden Valley Golf & Ski, Inc. o Facts: 16-year-old girl went skiing at midnight; went over mogul and suffered brain damage. Sued for negligence in maintaining the slope. o Issue: Alleges court erred in jury instructions, which said that verdict must be for if you believe conditions encountered were risks inherent in the sport of skiing. o Case is about Implied PRIMARY AOR: If youre hit by a foul ball in a baseball gamecomplete bar to liability- assumption of risk. Inherent in the sport of baseball. Exception today because of lawsuits- there is a net behind home plate to prevent being hit from a foul ball. Dont necessarily assume the risk of being hit by a foul ball. Hockey: Spectators assume risk of maybe being hit by a puck. Spectator on golf course: sued when he tripped over a rock in the ruff. Rocks on the course are inherent risk- golf is played outside. Skating: running into another skater- inherent.
79

o o o o

Rationale for protecting from liability: risk is what makes the game fun; if youre going to impose liability youre essentially saying you cant have that activity; injury is inherent risk of the sport. How do you define s conduct? Reasonable. ALSO had no duty to eliminate the inherent risk of skiing, or baseball, hockey, etc. Does it matter whether she was aware that there were moguls or not? allowed unrestricted access to the slopes. What if she was a novice and didnt know there were moguls if she went up to the intermediate levels? Court gave no weight to this. Did SHE voluntarily assume the risk? AOR is a subjective standard. If we describe it as a case a no duty case does it matter about her knowledge? No. Matters if you describe it as AOR or a duty. Most courts describe it as a no duty scenario. Exculpatory clauses deter these results, even though there is no duty it shouldnt matter that there is a waiver. A no-duty doctrine protects the organizers of & participants in a sporting or recreational activity from liability for injuries resulting from risks that are inherent in the activity- implied primary assumption of risk. The fact that the knowingly & voluntarily ran the risk that eventuated in harm does not itself justify barring recovery. If it was unreasonable to run the risk, the s recovery should be diminished or barred under the applicable comparative-fault principles. Hypothetical: Skydiving- doesnt sign exculpatory clause- implied primary AOR. Parachute doesnt open. What result? No duty? Could argue there WAS a duty- especially if chute was packed by the company. Duty because as the you rely on the s due care that it will open and be packed correctly. Different from skydiving, chute opens, and you break your ankle. This is inherent. Chute not opening isnt inherent and probably doesnt happen without someones negligence. This is a risk that CANT be eliminated from the sport or activity without changing the nature of the sport of the activity. Even if there was an exculpatory clause, it would probably be struck down for being against public policy. Foreigner at a baseball game, knows nothing of foul balls, can he recover? NO, no duty scenario- inherent risk of baseball. No duty to eliminate inherent risks- implied primary AOR. o Bright line rule, but there is an ambiguity- what defines an inherent risk? For AOR- you need to have a reasonable alternative. Like if you have a duty to partake in a dangerous activity or get fired- if you do the activity, its not an AOR. Today we have comparative negligence. Implied Secondary Unreasonable AOR overlaps with Comparative negligence, so today we just treat this as comparative negligence. How do you deal with implied secondary reasonable AOR?
80

Comparative negligence not here. In common law, it would be a complete bar to recovery. But now, do we continue to treat it more harshly than UNREASONABLE AOR? Some jurisdictions still treat it as no bar, others seeing the inconsistency treat it as comparative negligence- instruct jury on comparative- usually the effect is that the jury does not find comparative negligence because the acted reasonably. It could not reduce recovery at all.

AFFIRMATIVE DEFENSES: STATUTES OF LIMITATIONS & STATUTES OF REPOSE Statute of Limitations o SOL- statute that says has to bring his/her cause of action within a certain period of time; if he doesnt, can never bring the cause of action. o Policy behind SOL - suing within a period of time: Helps establish causation; Evidence is probably better, especially if you need eyewitnesses. Puts s on notice that they need to preserve evidence. Encourages s to get out there, investigate their claims, get evidence; Allows s be free of threat of suit. Overall purpose of SOL is to prevent stale claims based on old/bad evidence. Traditional rule regarding accrual of a cause of action: s cause of action typically accrues on the day of injury. o However it is very often the case today that this rule is modified by a DISCOVERY RULE. Discovery Rule: In most cases you dont need this. o Example- get in a car accident today, your car accident would accrue- you know all you need today to bring suit. You know youve been injured, you know there was negligence, you know the negligence caused your injury, you know the law provides you the right to sue, nothing to stop you from suing the next day. Typical case. o DES is not the typical case. Weird situation because injury occurs not to person who ingests the drug, but instead there is a very long latency period before the injury manifests. o The discovery rule provides that the accrual date of a cause of action is delayed until the is aware of her injury and its negligent cause. o A is held to her actual knowledge as well as knowledge that could be discovered through investigation of sources open to her. o Most SOLs talk about when cause of action accrues discovery rule helps modify/interpret SOLs in certain kinds of cases, particularly when a person is injured in an accident but the injuries dont manifest until later. Long period of latency (DES cases) o Equitable doctrine helping to interpret the SOLs. o The discovery rule is the most important deviation from the strict deadlines imposed by statutes of limitations. Jolly v. Eli Lilly & Co. o Mother given DES negligent act occurs = DATE OF THE NEGLIGENT CONDUCT 1951: born; latent injury that hasnt manifested itself yet. 1972: learns simultaneously that mom took DES and that diagnosed with adenosis. 1976: abnormal pap/ needs a dilation & curettage.
81

o o o

o o

1978: hysterectomy - never sues up to this point. Didnt sue because she had a causation problem- didnt know she could. Maybe didnt even know there was negligence- wasnt clear. 1980: Sindell decided market share liability theory way to get around causation problem. 1981: Suit filed. SOL in this case was one year. Her argument that SOL had not run was that Sindell revived her case because it gave her cause; brought a new element back. Made the cause of action legally available. Possible options for how discovery rule works: Modifies traditional rule about accrual for cause of action. Says accrual for cause of action is delayed until is aware at a minimum of her injury and it is often required they are aware of injury plus causedepends on jurisdiction. Discovery rule in CA: cause of action delayed until aware of her injury + negligent cause; however, ignorance of the legal significance of known facts or identity of does not delay accrual. What do s have to discover (or reasonably should have discovered) to start the running of a SOL? Option 1: injury Option 2: injury + factual cause (ex. DES) Option 3: injury + negligent cause (ex. Manufacturer) Jolly uses this option means cause of action accrues when you know you have been injured and the facts are such as to put you under suspicion that negligence was done by a is enough. Dont have to know the identity of the . Option 4: injury + cause + availability of legal remedy How different options would have worked in Jolly: Option 1- What date did begin to discover her injuries? 1972. If this were the triggering of the discovery rule, SOL would have begun to run in 1972 and would have been time barred in 1973. Option 2- what about discovery of injury plus factual cause? Learning that mom took DES is a factual cause. Time of action accrues in 1972 and is then time barred in 1973. Option 3- Not clear when she knows that negligence was caused; may know it was from her mom taking DES; but she may not have known it was due to someones tortious activity. Did she know or should she have known of the negligent cause of the ingestion of DES? Does KNOWING the negligent cause mean she also knows theres a VIABLE negligence action? Arguably not a viable action until 1980. Option 4- Flushes out negligent cause around a particular path- cause of action only accrues when you have an injury, know the cause, and know of available remedies. Is adenosis enough injury to bring suit? Pre-cancerous condition. Maybe actual injury doesnt manifest until years later. Have to have FILED suit within the period of SOL- suit doesnt have to be completed and discovery can go on for a long time. Court said Jolly was in the same situation as Sindell had same causation problem; court likely to throw out on summary judgment because cant establish causation.
82

o o

Court says Jolly should have gone forward with a novel theory of causation. Could have filed a Doe action and then conducted discovery between different drug manufacturers to figure out the cause. Even when she knew injury, cause, and that it was due to negligence, her claim was no good; but by the time Sindell came down it was still too late. Court said she should have started investigating earlier. Anytime you consult anyone and say they dont have a cause of action because of SOL you must tell them to act quickly to preserve their legal rights.

Continuing Tort Rule Another equitable doctrine. Continuing wrongful act; not just continuing effects. A continuing tort of violation is occasioned by continuing unlawful acts and conduct, not by continual ill effects from initial violation. A continuing tort does not involve tolling the statute of limitations because of delayed or continuing injuries, but instead involves viewing the s conduct as a continuous whole for perceptive purposes; a violation of the s rights was a continuous tort that did not cease until the date of the last injury or when the tortious act ceased. Feltmeier v. Feltmeier Domestic abuse case, even after divorce. She sued him for Intentional Infliction of Emotional Distress (Intentional Tort) Elements: Act Intent: Unique because in order to satisfy the mental state requirement you can have intent or recklessness in inflicting the emotional distress. (Conscious disregard of a high risk of causing conduct) Defining element of tort is the requirement of outrageous conduct. Outrageousness: Comes from Restatement (Second) 46: To judge outrageousness- the conduct has to be such as to make a reasonable person exclaim Outrageous! Severe emotional distress (Like physical manifestation requirement, has some leeway in how strictly or loosely courts require there to be severe emotional distress) Timeline: Married in 1986 Divorced in 1997 Files suit in August of 1999 SOL is 2 years. Husbands argument: He says he is not liable for conduct before 1997; says each act is separate incident of abuse; everything after 2-year SOL is barred. Only things after 1997 can be counted. He was trying to limit his liability. s argument: It was a continuous tort; overall course of conduct- under CT rule- he can be liable for a continuing course of conduct. Precedent cases: Ravenswood: Court said SOL ran on his claim after he brought suit for citys construction of a subway tunnel under the s property claiming it constituted a continuing trespass violation. Court said it had to be a continual wrongful act, not just a continual effect from the initial
83

violation. Needs to be a continual wrongful ACT, not just continuing EFFECTS. Belleville Toyota v. Toyota Motor Sales: s were sued for misconduct in the allocation of cars to dealership for a fair period of time; did this constitute continuous course of conduct for purpose of continuing tort rule? Court says no because every single time they made a conscious decision to do it; every time is a separate wrongful act. Discrete, separate act that is distinguishable from this case. If in this case sued husband for BATTERY, her case might have been ruled like Belleville; but it was the cumulative impact of the abuse that resulted in IIED that made her case distinguishable. Husband cited Belleville as the precedent case for his claim. Court found that s conduct as a WHOLE stated a cause of action for intentional infliction of emotional distress. IIED helps her get around SOL. Why doesnt discovery rule apply? - Rule says cause of action accrues when you discover your injury & cause, which she knew as early as 1986. The injury she is suing for is IIED, which arguably didnt manifest until years after abuse. Another argument is that both DISCOVERY RULE AND CONTINUOUS TORT RULES are equitable doctrines / ways of fixing unduly harsh results of SOLs that apply in certain circumstances. Dont apply them together- you pick one or the other!!!!!! Equitable doctrines to make it fair to the ! If you used them together it would negate the point of the doctrines.

Statutes of Repose Differ from SOLs Statutes of repose are a response to the possibility that despite SOLs, actionable claims may arise many years after the tortious act occurs. This can happen under any SOLs that do not begin to run until the injury occurs, and the problem is exacerbated by the discovery rule, which makes it possible for latediscovered claims to be brought many years after the injury. The statutes usually recognize no exceptions for incapacity or fraud, and their intent plainly is to provide an absolute deadline. o Bradway v. American Ntl. Red Cross Facts: gets surgery, contracts AIDS from blood transfusion. Issue: Is this a claim for ordinary negligence or for medical malpractice? Difference is the length of the SOLs. SOL for Ordinary Negligence: Actions for injures to the person shall be brought within 2 years after the cause of action accrues. SOL for Medical Malpractice: Statute (a) An action for medical malpractice shall be brought within 2 years after the date on which an injury or death arising from a negligent or wrongful act or omission occurred. Sounds the same, however, this is a statute of repose. GA Supreme Court- says this case is a medical malpractice claim because it involves medical services involving medical judgment. Therefore, have to apply the medical malpractice statute. Timeline April 1983: date of blood transfusion July 1988: manifestation of AIDS April 19, 1989: File suit - 6 years after transfusion
84

Is the injury AIDS or blood transfusion?? Statute of Repose: Notwithstanding (a), in no event may an action for medical malpractice be brought more than 5 years after the date on which the negligent or wrongful act or omission occurred. Therefore total bar to a claim after 5 years. Have to bring the claim within 2 years; but even if a discovery rule modifies the SOL making it a little longer, in NO EVENT may you bring a claim after 5 years. Discovery rule can make the claim be filed AFTER 2 years, but NO LONGER than 5!!!!! The ambiguity in statute about what the word accrual means - allows the judges to interpret this. Statutes of repose: give you longer periods of time before right to file claim is absolutely over. They express concern for the fact that some injuries are latent and dont manifest for a period of time. Also, its somewhat unfair to cut off peoples claims before they know they have an injury. SORs eliminate the long tail on claims- all claims after a certain date are barred. Lower insurance costs Dr.s perspective: doctors can breathe easy knowing period of possible suit is over. Damages are very high in med malpractice cases. Public policy reasons: Limits overall number of claims Encourages more doctors Protects medical profession, which is essential to the state. Could also prevent frivolous claims.

Exculpatory Clauses Look at LANGUAGE of clause and apply it strictly. Terms of waiver must cover kind of conduct that actually happened, or else its not relevant. Invalidating a clause on grounds of public policy: If you go to emergency room & they say they will only treat you if you agree to sign a waiver releasing any liability probably invalid on grounds of public policy. Medical/essential service- necessary hurdle to get treatment. Some hospitals do make you sign a waiver releasing claims over $100,000. Example: Cant sue Shands, doctors are independent contractors- can only sue them and not the hospital. Reasonable limits on the claim are more likely to be upheld than limits entirely. If they dont give you a reasonable time to read the waiver, it may be invalidated. Have to know SPECIFIC RISK- have to KNOW what you are signing. Incapacity: could maybe invalidate waiver. Interaction of tort & contract law.
85

STRICT LIABILITY o o o Intentional Torts Subjective Negligence Objective Strict Liability Instead of imposing a duty of reasonable care, it imposes a duty of absolute care. still has to prove CIF, proximate cause and damages. Policy issue.

Example: You decide to keep a tiger in your basement. You padlock the cage, and the basement is locked and has an alarm system. Tiger still gets loose and bites neighbor. Still liable even though youre not at fault. Inherently dangerous animal; reasonable people dont keep a tiger as a pet. o A reciprocal risk is when we impose risks upon another in society (like driving our cars); o Policy wise- notion of non reciprocal risks: Keeping a tiger poses a non reciprocal risk extra high degree of risk that through your choices you have chosen to impose on society without them posing a commensurate risk on you. o Strict liability provides an incentive to not impose a non reciprocal risk on society. Livestock Rule o Common law traditional rule with trespassing livestock: youre liable for the damage your livestock imposes on someone elses property or chattel. o Altered in Western states because it is relatively common in Western states- big economic activity. Shouldnt be strictly liable; thought it was too harsh. Negligencebased liability. o Fencing-In Statutes: have to fence in your animals. Burden is on rancher to fence in animals. o Fencing-Out Statutes: if you dont want animals on your land, burden on farmer to keep livestock out of his pasture. o Most states have fencing in statutes, burden on rancher. o Most economic/valuable player in state burden put on other party. Examples: o Big dog knocks down small child. Dog has bitten people before. Strict liability? No, could be just negligence. If youre on notice of violent propensities, could have a duty to use absolute care. o Tiger knocks down child, strict liability? Is this the type of harm that you are trying to protect? o Child comes to house; tiger sheds fur. Child sits on couch and has allergy attack and has to go to emergency room. Strict liability? Not the type of harm that you foresee. What makes you strictly liable is what is within the danger; foreseeable propensities of the tiger. o Comparative negligence (comparative responsibility) or AOR can sometimes be defenses to strict liability. o Harm that occurs must be something that makes the animal dangerous enough to justify imposing strict liability in the first place. o Cant compare fault and non-fault based conduct = so some jurisdictions dont allow comparative negligence.
86

Other jurisdictions let the jury assign the numbers and just call it comparative responsibility.

Rylands v. Fletcher o Two businesses whose interests are in conflict with each other; 1 has mine other has mill. o Coal mining shafts goes under mill owners property. He builds reservoir and it is not strong enough to support weight of water, breaks down and floods the disused mine shafts; which then flood the property of the mine owner. Mine owner sues. o Issue: Is mill owner responsible for damaging mine owners property? Yes. o Basis for imposing liability - Negligence? Mine owner may be comparatively negligent Independent contractors built the reservoir, not the actual mill owner. If its negligent, the mine owner may not be able to recover. So what is the basis for liability? o This case is the origin of strict liability for ABNORMALLY DANGEROUS ACTIVITY!!!!!! o Reasons for imposing strict liability - natural vs. non-natural or artificial use of property. Filing reservoir is artificial use of property. Ordinary or normal vs. abnormal Building artificial reservoirs in a coal-mining region not a normal or natural use of the land; threatens the more economically beneficial activity, therefore you are strictly liable. Turner v. Big Lake Oil Co. o Big Lake is an oil field town. Retention pond/reservoir filled with salt water which comes from injection wells. Salt water escaped and polluted a neighbors pond. Strict liability? No. This kind of reservoir in oil field country is essential to the economic viability to the region- very ordinary use of property in this region; normal use. Reservoir non-reciprocal risk In oil field country, it is a reciprocal risk because most people DO have these retention ponds. o Many times strict liability is NOT imposed to protect dominant industry in a city. But in oil field places, you can have reservoir which would otherwise be SL because it protects the entire industry. o Rylands- Does this only apply to injuries of the land? Yes. o Does it apply to artificial conditions as opposed to natural conditions? Now a predominantly historical interest; now it applies to a much broader category of classes. Siegler v. Kuhlman o Controversial case. o was driving a gasoline truck which flipped over and tumbled onto another highway below. No lights and another driver hit the truck and it went up in flames. Mystery over exactly why tanker of truck disengaged. All of the evidence was destroyed hard to prove breach. Need SL here because negligence is hard to prove. Could have used RIL to help prove breach. o Is a tanker coming loose from a trunk the type of thing that ordinary happens in the absence of negligence? Probably not. Most likely negligent party? Driver. Was it due to any action on the part of the plaintiff? Probably not.
87

o o o

Trial judge did not instruct RIL. Appellate court said trial court screwed up not giving a RIL instruction but it doesnt even matter basis they go off on is that this is a proper case for the imposition for strict liability and look to the Restatement to see if the factors show that this case is proper for it. Arguments for application of SL: driving a gas tanker is incredibly dangerous no matter how many precautions you take. Even when reasonable care is exercised, driving that quantity of gas has highly significant risk of physical harm. Arguments against SL on the facts of this case: How else do you transport gasoline? Matter of COMMON USAGE relevant fact!!!! Paradigm Case: Blasting with dynamite. Fairly common activity on construction sites. SL applied. Setting off fireworks SL Value to community is less for setting off fireworks than carrying gasoline. Non-reciprocal risk- this might be a disproportionate risk but we all benefit from it. Negligence liability is not enough- doesnt set any extra incentives. Reasonable care isnt enough. SL sets further incentives. SL over negligence liability might want the person engaged in the activity not to think solely about the risk but about where the appropriate location is to conduct the activity. Encourages maximum precautions. Sets structural incentives in a way reasonable care does not. Strong public policy desire to protect the innocent plaintiff. Concurrence: alternate rationale for applying SL- can spread the loss among customers. By making you liable for inevitable accidents, even with reasonable care, if forces you to internalize cost you impose on society. Raise price of gas it spreads the loss to the customers- people who benefit from the gasoline. Form of Enterprise Liability burden you impose on society & you should internalize the costs.

Restatement (Third) Section o Factors for applying strict liability for Abnormal Activity if: Activity creates foreseeable and highly significant risk of physical harm even when reasonable care exercised and Activity is not a matter of common usage Always argue common usage, case could then maybe go the other way. o Section 24: Unforeseeable intervening force will not cut off liability. Hypo: Blasting. Meat farmer on a neighboring farm to where blasting is taking place- claim that Ds blasting so frightens his meats that they eat their young. o What results? Still has to be foreseeable; blasting has an array of foreseeable risks attached to it. o Strict liability therefore not applied. o This is not why strict liability is imposed for blasting activity. o Outside the scope of the risk that makes blasting a strict liability activity. Strict liability = liability without fault NOT THE SAME as absolute liability.
88

o o

Absolute Liability: any injury from the use of the product youre liable. Doesnt matter if you caused it. Like Hymowitz cant get yourself out of liability even if you werent involved. Criticisms of market share D cant disprove fault; its absolute. Liability without fault but liability without causation as well. True absolute liability insurers liability. Allows no defenses based on the conduct of the plaintiffs are barred. No way out. If youre injured the insurer will pay you. Strict Liability: some defenses barred, others are not. Contemplates possibility that a defense may be possible based on plaintiffs conduct. Still have to prove causation and there may be defenses.

PRODUCTS LIABILITY Umbrella term that describes various theories of recovery. o Negligence o Breach of Warranty: contractual claim o Strict Products Liability: SL imposed on the basis that a product is defective. o Misrepresentation Chain of Distribution o Component parts manufacturer o Manufacturer o Distributor Retailer OR just from manufacturer to retailer o Purchaser (could be buying car and giving it as a gift to a USER) o User o Bystander (If wheel falls off, user injured & pedestrian, passenger, etc.) Negligently manufactured part can go through many hands before getting to injured person. Duty to anyone who could be foreseeably injured. Types of Defects: o 1. Manufacturing o 2. Design o 3. Warnings Privity of Contract Limitation (Eventually done away with) o Erosion of o Death of o Shift to strict liability for defective products Privity of K Limitation o Winterbottom vs. Wright stagecoach driver injured by negligent repair of stagecoach; he cant recover because he had no K with manufacturer, only OWNER of stagecoach could technically recover. o What if purchaser was injured? With whom is he in privity of K? RETAILER. Unless it was the retailer who was negligent, purchaser is out of luck.
89

Privity of K limits suit for negligence cant really go up chain to get person who was really negligent in a products liability case, unless you directly buy it from the manufacturer. Erosion of Privity Rule: o Thomas vs. Winchester: case about a pharmacy & medication preparation that was mislabeled. Purchaser- husband. Gives it to wife, wife is NOT in privity of K with negligent manufacturer of product, yet she is made violently ill and almost dies. o Broad exception created to privity rule: If its imminently or inherently dangerous!!!!! o Distinguish this from stagecoach case on the grounds that this is a case about a poisonous drug, harm is naturally and almost inevitably harmful if mislabeled. Death of Privity Rule o MacPherson v. Buick Motor Co.: 1916- Cardozo opinion; Buick purchased- had wooden wheel with spokes. o Wooden wheel broke & plaintiff injured. o Component parts manufacturer (of wheel); D- manufacturer of car; Retailer; Plaintiffinjured. o Does defendant owe a duty to the purchaser with whom it has no privity of K? o Cardozo says YES imminently dangerous exception not limited to poison; exception applies whenever there is a foreseeable and unreasonable risk of harm. Sounds like Palsgraf liable for anyone within the scope of the risk; foreseeable plaintiffs. o Privity of K officially gone as a limitation of duty. o Problem: Still need to be able to show exact negligent act; burden on plaintiff. o Hard to determine where negligent act happened. Res Ipsa? Downsides- hard to show who exactly had exclusive control of product. What if its obvious that the product is defective? o Some say if its obvious there is no liability. o Similar to assumption of risk. Shift to Strict Liability for Defective Products o Greenman v. Yuba Power Products, Inc. California 1963- Period of extending tort liability. Leading example of expansion of tort law. Shopsmith power tool that could be used as a saw, drill, and wood lathe. Person who was injured by this tool was NOT the purchaser; his wife bought it for him. Problem #1- no privity of K. Problem #2- injury occurs 2 years after it was purchased. Sues retailer (Sears) & the manufacturer (Yuba). Smart if youre going to sue for negligence gets everyone that could be liable involved in the suit. Husband studied the manufacturers brochure before the purchase. Relevance? Evidence that he used it correctly. Theories of liability he pursues: Negligence & breach of warranty theories. Success: Didnt work against retailer, but worked against manufacturerjury said manufacturer not liable for breach of implied warranty, but jury said they were liable for express breach of warranty.
90

Court imposes strict liability in tort because of the problems with existing theories of liability. Warranty law as source of recovery- problems based on K doesnt give best remedies. Manufacturer tried to argue warranty claim was no good because of express warranty- not in privity of K. Warranty law: extends beyond purchaser to foreseeable user of product. Another limitation: notice requirements of warranty law- have to give proper notification within a period of time by contract that youre planning to file a warranty claim. They argue he didnt do this. But Court says this is ridiculous because hes not the one that bought the product. Problems with negligence law- proof products going up the chain. Appropriate situation to apply strict liability not in contract, but in tort. Holding of case: Have to prove youre using the item in the way that it was intended to be used. Today foreseeable misuses dont deny recovery. i.e. Prying something open with a screwdriver. Essence of liability in strict liability cases: Defining characteristic: Allocates the cost; cost internalization. Manufacturer places in the stream of commerce a defective product and is known to cause injury to human beings. Hallmark of strict products liability is defectiveness. How products are marketed: Way the consumer expected product to be. If they are marketed for a certain use there is an implied representation of safety that comes with the product. Significance of this case: leader of imposing strict products liability.

Rationales o Compensation (loss spreading) = also in negligence o Deterrence = also in negligence o Cost internalization= also in negligence o Proof problems unique rationale for strict liability o Consumer expectations unique rationale for strict liability o Strict liability = if youre in the business of producing the product, you can internalize cost and pass it onto society. You impose costs on society and therefore the people who buy product ought to bear the true cost your product imposes. Very similar to negligence = risk utility test vs. negligence test Essence of negligence liability: reasonableness; failure to behave as a reasonable person o Essence of strict liability: defectiveness operative underlying concept Focuses on condition of the product Did it differ from its intended design? Dont have to prove negligence . Could potentially use RIL to prove negligence, but there are limits: o Easier for manufacturer to defend- defect could have happened at another level in the hierarchy. o Maybe not under exclusive control.

91

Example: Hair line fracture in coke bottle could have happened at any stage in the line. RIL hard to prove where it happened. Strict products liability works much easier.

Types of Defects: o Manufacturing Defect: test is if it looks different; visibility- departs from intended design Example: Auto sent to consumer with lug nuts hanging loose in wheel. o Design Defect: All products in the line have defects. Planned it to be that way. Made choice to not have features that would arguably have made it safer. CHOICE to not put mirrors on crane. This choice made it unreasonably safe for consumers. o Warnings Defect: Lack of warnings or deficiency in warnings that render a product unreasonably dangerous to the reasonable user of product/consumer. Must warn of intended use or reasonably foreseeable misuses. Example: Weed Killer without a warning that it has extreme toxicity with skin contact. Test for Design Defect: o Consumer Expectations Test: Restatement Second Very influential- defining test for products liability (1) People subject to strict products liability according to test: One who sells... One who is in the BUSINESS of selling products- not a person who sells at a garage sale, or people who sell you a service where the use of a product is incidental to the service. o Go to hairdresser where you get hair dyed- service or selling of a product? Seller of product: Manufacturer, Distributor, and Retailer o Retailer: Sears o Manufacturer: Company A o Sold to consumer at Sears. Sears is liable if product is defective. o Have responsibility to inspect products they sell to their customers. o Easier for purchaser or user to know who to go against. Mitigating Doctrine: If Sears is liable for defect, Sears can go up the chain and seek indemnification for what it had to pay out. Can get its money back from manufacturer. Works a little like joint & several liability make sure there is a source of money for the consumer (person injured) to recovery from. All of the rationales for applying strict liability apply to Sears as it does to the manufacturer. (2) Applies to user or consumer even if they didnt buy the product. What about the bystander? Many courts have extended liability to the foreseeable injured bystander. Test for defectiveness - Consumer Expectations Test: o Dangerously beyond expectations of ordinary consumer. Implied representation of safety. Defective condition unreasonably dangerous- can be defective that is not dangerous. Defect must make product unreasonably dangerous. P must establish that it is in an unreasonably dangerous defective condition. Has to reach consumer without substantial change
92

Cant be due to any action of the consumer

Relevance of Knowledge of Defect: o Negligence Relevance of knowledge of manufacturer of defects- liable if you knew- failure to behave reasonably. Reasonable manufacturers = knowledge that they should have had Relevant what they knew or should have known taken into account what was foreseeable at that time. Look at conduct of manufacturer with foresight of potential danger o Strict Products Liability: Knowledge of manufacturer is irrelevant; as is what they should have known Obvious Feature: o Gray v. Manitowoc Co.: Consumer Expectations Test When open & obvious product doesnt meet consumer expectations exception P knew there was a blind side to the crane. Happened twice. Not definitive whether he knew it was the blind side of the crane look what reasonable people in the industry know. Objective standard for judging obviousness. They had flagmen to remedy the dangerousness of the product. Objectively reasonable person would have avoided blind side of crane, and he actually knew. Open & obvious defect bars recovery completely. No reasonable jury could have concluded the crane was dangerous to a degree not anticipated by the ordinary consumer of that product. If obviousness is contested it goes to the jury. Phillips v. Kimwood Machine Co. Different test for defectiveness. Strict liability risk utility test importance of showing a reasonable alternative design How does this test deviate from negligence liability test?

93

94

Вам также может понравиться